2410 Exam 1

Pataasin ang iyong marka sa homework at exams ngayon gamit ang Quizwiz!

A patient has dehydration. While planning care, the nurse considers that the majority of the patient's total water volume exists in with compartment? a. Intracellular b. Extracellular c. Intravascular d. Transcellular

a. Intracellular

A client in the emergency department reports extreme dry mouth, constipation, and an inability to void. The client's history includes incontinence. Which question by the nurse is most important? a. "Are you drinking plenty of water?" b. "Do you take anticholinergic medication?" c. "Have you tried laxatives or enemas?" d. "Has this type of thing ever happened before?"

b. "Do you take anticholinergic medication?"

Which finding in an analysis of cerebrospinal fluid (CSF) is consistent with a diagnosis of bacterial meningitis? a. CSF appears cloudy. b. CSF pressure is decreased. c. Few leukocytes are present. d. Glucose level is increased compared with blood.

a. CSF appears cloudy.

caring for a client in the hyperdynamic phase of septic shock. Which medication does the nurse expect to be prescribed? a. Heparin sodium b. Vitamin K c. Corticosteroids d. Hetastarch (Hespan)

a. Heparin sodium

The mother of an HIV-positive infant who is 2 months old questions the nurse about which childhood immunizations her child will be able to receive. Which immunizations should an HIV-positive child be able to receive according to the American Academy of Pediatrics recommendation for immunizing infants who are HIV positive? Select all that apply. a. Hepatitis B b. DTaP c. MMR d. IPV e. HIB

a. Hepatitis B b. DTaP d. IPV e. HIB

The patient and the nurse are discussing Rickettsia rickettsii—Rocky Mountain spotted fever. Which patient statement to the nurse indicates understanding regarding the mode of transmission for this disease? a. "When camping, I will use sunscreen." b. "When camping, I will drink bottled water." c. "When camping, I will wear insect repellent." d. "When camping, I will wash my hands with hand gel."

c. "When camping, I will wear insect repellent."

A client is undergoing radiation therapy and asks the nurse about skin care for the exposed area. Which statement by the nurse is most accurate? a. "No products work well to reduce the skin reactions you get from radiation." b. "No one product works best, so you can choose what you would like to use." c. "The only medication that works well for skin reactions is very expensive." d. "No good studies on skin care with radiation have been conducted to date."

b. "No one product works best, so you can choose what you would like to use."

The nurse is caring for a group of patients. Which patient will the nurse see first? a. A patient with D5W hanging with the blood b. A patient with type A blood receiving type O blood c. A patient with intravenous potassium chloride that is diluted d. A patient with a right mastectomy and an intravenous site in the left arm

a. A patient with D5W hanging with the blood

A nurse is teaching a health class about colorectal cancer. Which information should the nurse include in the teaching session? (Select all that apply.) a. A risk factor is smoking. b. A risk factor is high intake of animal fats or red meat. c. A warning sign is rectal bleeding. d. A warning sign is a sense of incomplete evacuation. e. Screening with a colonoscopy is every 5 years, starting at age 50. f. Screening with flexible sigmoidoscopy is every 10 years, starting at age 50.

a. A risk factor is smoking. b. A risk factor is high intake of animal fats or red meat. c. A warning sign is rectal bleeding. d. A warning sign is a sense of incomplete evacuation.

What should the nurse recognize as symptoms of a brain tumor in a school-age child for whom she is caring? Select all that apply. a. Blurred vision b. Increased head circumference c. Vomiting when getting out of bed d. Intermittent headache e. Declining academic performance

a. Blurred vision c. Vomiting when getting out of bed d. Intermittent headache e. Declining academic performance

A client with advanced cancer is being treated with intravenous mithramycin (Mithracin). Which clinical manifestation indicates that the treatment is effective? a. Bowel sounds are active in all four quadrants. b. The client's serum sodium level is 138 mEq/L. c. The pulse rate is 68 beats/min and bounding. d. Urine output has increased to 30 mL/hr.

a. Bowel sounds are active in all four quadrants.

The nurse is planning a cancer education event in an Asian community center. The nurse plans to present information specifically on which types of cancer? a. Breast and colorectal b. Skin and lymphoma c. Liver and stomach d. Uterine and ovarian

a. Breast and colorectal

When discussing estrogen replacement therapy (ERT) with a perimenopausal woman, the nurse should include the risks of a. Breast cancer b. Vaginal and urinary tract atrophy c. Osteoporosis d. Arteriosclerosis

a. Breast cancer

A nurse is providing follow-up care for cancer survivors. Which condition should the nurse most monitor for in these patients? a. Cancer b. Infection c. Weight gain d. Low blood pressure

a. Cancer

A nurse is assessing an infant for urinary tract infection (UTI). Which assessment findings should the nurse expect? Select all that apply. a. Change in urine odor or color b. Enuresis c. Fever or hypothermia d. Voiding urgency e. Poor weight gain

a. Change in urine odor or color c. Fever or hypothermia e. Poor weight gain

A nurse is selecting a site to insert an intravenous (IV) catheter on an adult. Which actions will the nurse take? (Select all that apply.) a. Check for contraindications to the extremity. b. Start proximally and move distally on the arm. c. Choose a vein with minimal curvature. d. Choose the patient's dominant arm. e. Select a vein that is rigid. f. Avoid areas of flexion.

a. Check for contraindications to the extremity. c. Choose a vein with minimal curvature. f. Avoid areas of flexion.

The nurse is seeing clients in a clinic. Which client does the nurse assess further for the development of cancer? a. Client with a cough that has lasted for 4 months b. Client whose mother died of lung cancer c. Client with a 10-pound weight gain d. Woman whose last mammogram was 3 years ago

a. Client with a cough that has lasted for 4 months

The nurse is working in an incontinence clinic and sees older clients. The nurse plans a habit training program for the client with which condition? a. Confusion b. Diabetes c. Early kidney failure d. Arthritis

a. Confusion

A female patient complains of a "scab that just won't heal" under her left breast. During your conversation, she also mentions chronic fatigue, loss of appetite, and slight cough, attributed to allergies. What are the nurse's next steps? a. Continue to conduct a symptom analysis to better understand the patient's symptoms and concerns. b. End the appointment and tell the patient to use skin protection during sun exposure. c. Suggest further testing with a cancer specialist and provide the appropriate literature. d. Tell her to put a bandage on the scab and set a follow-up appointment in one week.

a. Continue to conduct a symptom analysis to better understand the patient's symptoms and concerns.

A client has septic shock. Which hemodynamic parameters does the nurse correlate with this type of shock? (Select all that apply.) a. Decreased cardiac output b. Increased cardiac output c. Increased blood glucose d. Decreased blood glucose e. Increased serum lactate f. Decreased serum lactate

a. Decreased cardiac output c. Increased blood glucose e. Increased serum lactate

The nurse reads in the medical record that a client has Kussmaul respirations. Which assessment finding is consistent with this condition? a. Deep, rapid respirations b. Respirations with an irregular pattern c. Shallow, grunting respirations d. Use of accessory muscles when breathing

a. Deep, rapid respirations

Which action is the primary concern in the treatment plan for a child with persistent vomiting? a. Detecting the cause of vomiting b. Preventing metabolic acidosis c. Positioning the child to prevent further vomiting d. Recording intake and output

a. Detecting the cause of vomiting

The nurse and the student nurse are caring for two different patients on the medical-surgical unit. One patient is in airborne precautions, and one is in contact precautions. The nurse explains to the student different interventions for care. Which information will the nurse include in the teaching session? (Select all that apply.) a. Dispose of supplies to prevent the spread of microorganisms. b. Wash hands before entering and leaving both of the patients' rooms. c. Be consistent in nursing interventions since there is only one difference in the precautions. d. Apply the knowledge the nurse has of the disease process to prevent the spread of microorganisms. e. Have patients in airborne precautions wear a mask during transportation to other departments. f. Check the working order of the negative-pressure room for the airborne precaution patient on admission and at discharge.

a. Dispose of supplies to prevent the spread of microorganisms. b. Wash hands before entering and leaving both of the patients' rooms. d. Apply the knowledge the nurse has of the disease process to prevent the spread of microorganisms. e. Have patients in airborne precautions wear a mask during transportation to other departments.

A client who has a local infection of the right forearm is being discharged. The nurse teaches the client to seek immediate medical attention if which complication occurs? a. Dizziness on changing position b. Increased urine output c. Warmth and redness at site d. Low-grade temperature

a. Dizziness on changing position

A benign breast condition that includes dilation and inflammation of the collecting ducts is called a. Ductal ectasia b. Intraductal papilloma c. Chronic cystic disease d. Fibroadenoma

a. Ductal ectasia

Which assessments will alert the nurse that a patient's IV has infiltrated? (Select all that apply.) a. Edema of the extremity near the insertion site b. Reddish streak proximal to the insertion site c. Skin discolored or pale in appearance d. Pain and warmth at the insertion site e. Palpable venous cord f. Skin cool to the touch

a. Edema of the extremity near the insertion site c. Skin discolored or pale in appearance f. Skin cool to the touch

The nurse is caring for a postoperative client who suddenly reports difficulty breathing and sharp chest pain. After notifying the Rapid Response Team, what is the nurse's priority action? a. Elevate the head of the bed and apply oxygen. b. Listen to the client's lung sounds. c. Pull the call bell out of the wall socket. d. Assess the client's pulse oximetry.

a. Elevate the head of the bed and apply oxygen.

The nurse is assessing a client who is in early stages of hypovolemic shock. Which manifestations does the nurse expect? (Select all that apply.) a. Elevated heart rate b. Elevated diastolic blood pressure c. Decreased body temperature d. Elevated respiratory rate e. Decreased pulse rate

a. Elevated heart rate b. Elevated diastolic blood pressure d. Elevated respiratory rate

A nurse should expect which cerebral spinal fluid (CSF) laboratory results on a child diagnosed with bacterial meningitis? Select all that apply. a. Elevated white blood count (WBC) b. Decreased protein c. Decreased glucose d. Cloudy in color e. Increase in red blood cells (RBC)

a. Elevated white blood count (WBC) c. Decreased glucose e. Increase in red blood cells (RBC)

The nurse is planning care for a client with late-phase septic shock. All of the following treatments have been prescribed. Which prescription does the nurse question? a. Enoxaparin (Lovenox) 40 mg subcutaneous twice daily b. Transfusion of 2 units of fresh frozen plasma c. Regular insulin intravenous drip per protocol d. Cefazolin (Ancef) 1 g IV every 6 hours

a. Enoxaparin (Lovenox) 40 mg subcutaneous twice daily

A client is receiving high-dose chemotherapy for multiple myeloma. Which intervention is most important for the nurse to implement to prevent complications during chemotherapy? a. Ensure that the client's fluid intake is 3000 to 5000 mL/day. b. Monitor telemetry every hour during therapy. c. Apply pressure to all injection sites for 5 minutes. d. Assist the client in all ambulatory activities.

a. Ensure that the client's fluid intake is 3000 to 5000 mL/day.

The exact cause of breast cancer remains undetermined. Researchers have found that there are a number of common risk factors that increase a woman's chance of developing a malignancy. It is essential for the nurse who provides care to women of any age to be aware of which risk factors? Select all that apply. a. Family history b. Late menarche c. Early menopause d. Race e. Nulliparity or first pregnancy after age 30

a. Family history d. Race e. Nulliparity or first pregnancy after age 30

A nurse is assessing a patient who is receiving a blood transfusion and finds that the patient is anxiously fidgeting in bed. The patient is afebrile and dyspneic. The nurse auscultates crackles in both lung bases and sees jugular vein distention. On which transfusion complication will the nurse focus interventions? a. Fluid volume overload b. Hemolytic reaction c. Anaphylactic shock d. Septicemia

a. Fluid volume overload

The nurse administers an intravenous (IV) hypertonic solution to a patient. In which direction will the fluid shift? a. From intracellular to extracellular b. From extracellular to intracellular c. From intravascular to intracellular d. From intravascular to interstitial

a. From intracellular to extracellular

In evaluating dietary teaching for a client with chemotherapy-induced neutropenia, the nurse becomes concerned when the client makes which food choice? a. Fruit salad b. Applesauce c. Steamed broccoli d. Baked potato

a. Fruit salad

Which home care instructions should the nurse provide to the parents of a child with acquired immunodeficiency syndrome (AIDS)? Select all that apply. a. Give supplemental vitamins as prescribed. b. Yearly influenza vaccination should be avoided. c. Administer trimethoprim-sulfamethoxazole (Bactrim) as prescribed. d. Notify the physician if the child develops a cough or congestion. e. Missed doses of antiretroviral medication do not need to be recorded.

a. Give supplemental vitamins as prescribed. c. Administer trimethoprim-sulfamethoxazole (Bactrim) as prescribed. d. Notify the physician if the child develops a cough or congestion.

The nurse is caring for a young woman with breast cancer. The stress between the woman and spouse is obvious, as is anxiety among the children. What is the nurse's best action in this situation? a. Help find or develop an educational program for the patient and spouse. b. Encourage the patient to agree with the spouse. c. Support the spouse, and explain that the spouse knows what is best. d. Take the children away and recommend foster care.

a. Help find or develop an educational program for the patient and spouse.

Which response is an example of compensation for an acid-base imbalance? a. Increase in the rate and depth of respirations when exercising b. Increased urinary output when blood pressure increases during exercise c. Increased thirst when spending time in an excessively dry environment d. Increased release of acids from kidneys during exacerbation of chronic obstructive pulmonary disease (COPD)

a. Increase in the rate and depth of respirations when exercising

The nurse is planning care for a client with hypercalcemia secondary to bone metastasis. Which interventions are included in the plan? (Select all that apply.) a. Increase oral fluids. b. Place an oral airway at the bedside. c. Monitor for Chvostek's sign. d. Implement seizure precautions. e. Assess for hyperactive reflexes. f. Observe for muscle weakness.

a. Increase oral fluids. f. Observe for muscle weakness.

In a client with less than the normal amount of bicarbonate in the blood and other extracellular fluids, what response does the nurse anticipate? a. Increased risk for acidosis b. Decreased risk for acidosis c. Increased risk for alkalosis d. Decreased risk for alkalosis

a. Increased risk for acidosis

9. The nurse correlates "initiation" in cancer development with which action? a. Inflicting mutations that lead to excessive cell division b. Increasing the capacity of the transformed cell for error-free DNA repair c. Stimulating contact inhibition in cells damaged by a carcinogen d. Making cancer cells appear more normal to escape immune surveillance

a. Inflicting mutations that lead to excessive cell division

The nurse is caring for a client who has a sealed radiation implant for cervical cancer. Which activities by the nurse are appropriate? (Select all that apply.) a. Inform the supervisor of the nurse's positive pregnancy test. b. Obtain the dosimeter badge from the nurse going off shift. c. Keep the client's door open for frequent observation. d. Dispose of dirty linen in a red "biohazard" bag. e. Wear a lead apron while providing client care.

a. Inform the supervisor of the nurse's positive pregnancy test. e. Wear a lead apron while providing client care.

Which nursing intervention should not be included in the postoperative plan of care for a child undergoing surgery for a brain tumor? a. Place the child in Trendelenburg position. b. Perform neurologic assessments. c. Assess dressings for drainage. d. Monitor temperature.

a. Place the child in Trendelenburg position.

The nurse notes that a child's gums bleed easily and he has bruising and petechiae on his extremities. What laboratory values are consistent with these symptoms? a. Platelet count of 19,000/mm3 b. Prothrombin time of 11 to 15 seconds c. Hematocrit of 34 d. Leukocyte count of 14,000/mm3

a. Platelet count of 19,000/mm3

In the client with alkalosis, the nurse assesses for which clinical manifestations? (Select all that apply.) a. Positive Chvostek's sign b. Positive Trousseau's sign c. Hyporeflexia d. Bradycardia e. Elevated blood pressure f. Elevated urinary output

a. Positive Chvostek's sign b. Positive Trousseau's sign d. Bradycardia

While planning care for a patient experiencing fatigue due to chemotherapy, which of the following is the most appropriate nursing intervention? a. Prioritization and administration of nursing care throughout the day b. Completing all nursing care in the morning so the patient can rest the remainder of the day c. Completing all nursing care in the evening when the patient is more rested d. Limiting visitors, thus promoting the maximal amount of hours for sleep

a. Prioritization and administration of nursing care throughout the day

The nurse has received a report from the emergency department that a patient with tuberculosis will be coming to the unit. Which items will the nurse need to care for this patient? (Select all that apply.) a. Private room b. Negative-pressure airflow in room c. Surgical mask, gown, gloves, eyewear d. N95 respirator, gown, gloves, eyewear e. Communication signs for droplet precautions f. Communication signs for airborne precautions

a. Private room b. Negative-pressure airflow in room d. N95 respirator, gown, gloves, eyewear f. Communication signs for airborne precautions

A nurse is caring for a patient with peripheral intravenous (IV) therapy. Which task will the nurse assign to the nursing assistive personnel? a. Recording intake and output b. Regulating intravenous flow rate c. Starting peripheral intravenous therapy d. Changing a peripheral intravenous dressing

a. Recording intake and output

A client has moderate acidosis. Which assessment does the nurse perform first? a. Take the client's pulse and blood pressure, and analyze the electrocardiogram (ECG) strip. b. Assess respiratory rate and depth and work of breathing. c. Perform assessments of musculoskeletal strength. d. Determine whether the client is awake, alert, and oriented.

a. Take the client's pulse and blood pressure, and analyze the electrocardiogram (ECG) strip.

The nurse teaches a client with superior vena cava syndrome that improvement is characterized by which clinical manifestation? a. The client's hands are less swollen. b. Breath sounds are clear bilaterally. c. The client's back pain is relieved. d. Pedal edema is present.

a. The client's hands are less swollen.

Which statement is true regarding how infants acquire immunity? a. The infant acquires humoral and cell-mediated immunity in response to infections and immunizations. b. The infant acquires maternal antibodies that ensure immunity up to 12 months age. c. Active immunity is acquired from the mother and lasts 6 to 7 months. d. Passive immunity develops in response to immunizations.

a. The infant acquires humoral and cell-mediated immunity in response to infections and immunizations.

The parents of a child with acid-base imbalance ask the nurse about mechanisms that regulate acid-base balance. Which statement by the nurse accurately explains the mechanisms regulating acid-base balance in children? a. The respiratory, renal, and chemical-buffering systems b. The kidneys balance acid; the lungs balance base c. The cardiovascular and integumentary systems d. The skin, kidney, and endocrine systems

a. The respiratory, renal, and chemical-buffering systems

When an adolescent with a new diagnosis of Ewing sarcoma asks the nurse about treatment, the nurse's response is based on the knowledge that (select all that apply) a. This type of tumor invades the bone. b. Management includes chemotherapy, surgery, and radiation. c. Ewing sarcoma is usually not responsive to either chemotherapy or radiation. d. Affected bones such as ribs and proximal fibula may be removed to excise the tumor. e. Is the most common bone tumor seen in children.

a. This type of tumor invades the bone. b. Management includes chemotherapy, surgery, and radiation. d. Affected bones such as ribs and proximal fibula may be removed to excise the tumor.

The nurse is dressed and is preparing to care for a patient in the perioperative area. The nurse has scrubbed hands and has donned a sterile gown and gloves. Which action will indicate a break in sterile technique? a. Touching clean protective eyewear b. Standing with hands above waist area c. Accepting sterile supplies from the surgeon d. Staying with the sterile table once it is open

a. Touching clean protective eyewear

Individuals of low socioeconomic status are at an increased risk for infection because of which of the following? (Select all that apply.) a. Uninsured or underinsured status b. Easy access to health screenings c. High cost of medications d. Inadequate nutrition

a. Uninsured or underinsured status c. High cost of medications d. Inadequate nutrition

The nurse is making a home visit to a child who has a chronic disease. Which finding has the greatest implication for acid-base aspects of this patient's care? a. Urine output is very small today. b. Whites of the eyes appear more yellow. c. Skin around the mouth is very chapped. d. Skin is sweaty under three blankets.

a. Urine output is very small today.

A client has bone cancer. What intervention does the nurse implement as a priority for this client? a. Using a lift sheet when repositioning the client b. Positioning the client's heels to keep them from touching the mattress c. Providing small, frequent meals rich in calcium and phosphorus d. Applying pressure for 5 minutes after intramuscular injections

a. Using a lift sheet when repositioning the client

Which assessment findings indicate to the nurse that a child has excess fluid volume? Select all that apply. a. Weight gain b. Decreased blood pressure c. Moist breath sounds d. Poor skin turgor e. Rapid bounding pulse

a. Weight gain c. Moist breath sounds e. Rapid bounding pulse

A client with prostate cancer says that he is now having a lot of pain in his lower back and legs. The nurse educates the client about which intervention? a. X-rays of the spine and legs b. Administering ibuprofen (Motrin) for pain c. Referral to the pain control specialist d. Referral to physical therapy

a. X-rays of the spine and legs

The nurse is caring for a diabetic patient in renal failure who is in metabolic acidosis. Which laboratory findings are consistent with metabolic acidosis? a. pH 7.3, PaCO2 36 mm Hg, HCO3- 19 mEq/L b. pH 7.5, PaCO2 35 mm Hg, HCO3- 35 mEq/L c. pH 7.32, PaCO2 47 mm Hg, HCO3- 23 mEq/L d. pH 7.35, PaCO2 40 mm Hg, HCO3- 25 mEq/L

a. pH 7.3, PaCO2 36 mm Hg, HCO3- 19 mEq/L

A postmenopausal female client has had two episodes of bacterial urethritis in the last 6 months. She asks her nurse why this is happening to her now. Which is the nurse's best response? a. "Your immune system becomes less effective as you age." b. "Low estrogen levels can make the tissue more susceptible to infection." c. "You should be more careful with your personal hygiene in this area." d. "It is likely that you have an untreated sexually transmitted infection."

b. "Low estrogen levels can make the tissue more susceptible to infection."

The patient had diarrhea for 5 days and developed an acid-base imbalance. Which statement would indicate that the nurse's teaching about the acid-base imbalance has been effective? a. "To prevent another problem, I should eat less sodium during diarrhea." b. "My blood became too acid because I lost some base in the diarrhea fluid." c. "Diarrhea removes fluid from the body, so I should drink more ice water." d. "I should try to slow my breathing so my acids and bases will be balanced."

b. "My blood became too acid because I lost some base in the diarrhea fluid."

A client's cancer is staged by the TNM classification as T1, N3, M1. What is the nurse's interpretation of this classification? a. The client has a large tumor involving the lymph nodes, but no distant metastasis. b. The client has a tumor, and metastasis cannot be determined by the staging method. c. The client has a 2-cm tumor, one involved lymph node, and local metastasis. d. The client has a small tumor, many involved lymph nodes, and distant metastasis.

d. The client has a small tumor, many involved lymph nodes, and distant metastasis.

The nurse is providing health education to a client on immunosuppressant therapy. Which instructions does the nurse include in this client's teaching? (Select all that apply.) a. "Wear a facemask at all times." b. "Take your temperature once a day." c. "Drink only bottled water." d. "Avoid any contact with pets." e. "Wash dishes with hot sudsy water." f. "Rinse your toothbrush in liquid laundry bleach."

b. "Take your temperature once a day." e. "Wash dishes with hot sudsy water." f. "Rinse your toothbrush in liquid laundry bleach."

A client is beginning to undergo urinary bladder training. Which is an effective instruction to give this client? a. "Use the toilet at the first urge, rather than at specific intervals." b. "Try to consciously hold your urine until the scheduled toileting time." c. "Initially try to use the toilet at least every half-hour for 24 hours." d. "The toileting interval can be increased once you have been continent for 1 week."

b. "Try to consciously hold your urine until the scheduled toileting time."

A client's spouse reports that the last time the client received lorazepam (Ativan) before receiving chemotherapy, the client was extremely drowsy and didn't remember the trip home. Which is the nurse's best action? a. Hold the dose of lorazepam for this round of chemotherapy. b. Explain that this is a normal response to the drug. c. Perform a Mini-Mental State Examination. d. Document the response in the client's chart.

b. Explain that this is a normal response to the drug.

The nurse is providing health promotion education to a client who has a family history of leukemia. Which factor does the nurse teach this client to avoid? a. Alcohol consumption b. Exposure to ionizing radiation c. High-cholesterol diet d. Smoking cigarettes

b. Exposure to ionizing radiation

The nurse is providing discharge teaching. Which statement by the client indicates the need for further teaching regarding increased risk for metabolic alkalosis? a. "I don't drink milk because it gives me gas and diarrhea." b. "I have been taking digoxin every day for the last 15 years." c. "I take sodium bicarbonate after every meal to prevent heartburn." d. "In hot weather, I sweat so much that I drink six glasses of water each day."

c. "I take sodium bicarbonate after every meal to prevent heartburn."

The client is scheduled for a colonoscopy. Which statement indicates that the client needs additional teaching about the procedure? a. "I may have gas and abdominal cramps after the test." b. "I will take strong laxatives the afternoon before the test." c. "I will take my Coumadin with a sip of water tomorrow morning." d. "I will take nothing by mouth after midnight on the day of the test."

c. "I will take my Coumadin with a sip of water tomorrow morning."

A client states, "At night, I usually need to sleep propped up on two pillows in the chair, but now it seems I need three pillows." What is the nurse's best response? a. "You should try to rest more during the day." b. "You should try to lie flat for short periods of time." c. "You need to stay in the hospital for further evaluation." d. "You can take medication at night so you can sleep."

c. "You need to stay in the hospital for further evaluation."

A client is receiving treatment with liquid nitrofurantoin (Furadantin). Which is the highest priority instruction that the nurse can provide to this client regarding accurate administration of the medication? a. "The medication should be mixed with cold water before drinking it." b. "Urine will turn orange immediately after you swallow the drug." c. "You should ask the pharmacist for a syringe to measure the dose." d. "The drug is available in granules that must be dissolved."

c. "You should ask the pharmacist for a syringe to measure the dose."

A patient presents to the emergency department with reports of vomiting and diarrhea for the past 48 hours. The health care provider orders isotonic intravenous (IV) therapy. Which IV will the nurse prepare? a. 0.225% sodium chloride (1/4 NS) b. 0.45% sodium chloride (1/2 NS) c. 0.9% sodium chloride (NS) d. 3% sodium chloride (3% NaCl)

c. 0.9% sodium chloride (NS)

A patient is to receive 1000 mL of 0.9% sodium chloride intravenously at a rate of 125 mL/hr. The nurse is using microdrip gravity drip tubing. Which rate will the nurse calculate for the minute flow rate (drops/min)? a. 12 drops/min b. 24 drops/min c. 125 drops/min d. 150 drops/min

c. 125 drops/min

A nurse has just received a bag of packed red blood cells (RBCs) for a patient. What is the longest time the nurse can let the blood infuse? a. 30 minutes b. 2 hours c. 4 hours d. 6 hours

c. 4 hours

The nurse is assessing clients in the emergency department. Which client is at highest risk for developing septic shock? a. 25-year-old man who has irritable bowel syndrome b. 37-year-old woman who is 20% above ideal body weight c. 68-year-old woman who is being treated with chemotherapy d. 82-year-old man taking beta blockers for hypertension

c. 68-year-old woman who is being treated with chemotherapy

A client has an arterial blood gas pH of 7.48. How does the nurse interpret this client's acid-base status? a. An unknown acid-base balance status b. A normal blood hydrogen ion concentration c. A deficit in blood hydrogen ion concentration d. An excess in blood hydrogen ion concentration

c. A deficit in blood hydrogen ion concentration

A nurse is teaching parents about the importance of immunizations for infants because of immaturity of the immune system. The parents demonstrate that they understand the teaching if they make which statement? a. "The spleen reaches full size by 1 year of age." b. "IgM, IgE, and IgD levels are high at birth." c. "IgG levels in the newborn infant are low at birth." d. "Absolute lymphocyte counts reach a peak during the first year."

d. "Absolute lymphocyte counts reach a peak during the first year."

The nurse is caring for a client who has had an anaphylactic event. Which priority question does the nurse ask to determine whether the client is experiencing distributive shock? a. "Is your blood pressure higher than usual?" b. "Are you having pain in your throat?" c. "Have you been vomiting?" d. "Are you usually this swollen?"

d. "Are you usually this swollen?"

The nurse expects to find renal compensation for an acid-base imbalance in which situation? a. Mild to moderate dehydration in a middle-aged client who jogged for 2 hours b. Acute asthma attack with wheezing of 6 hours' duration in an older man c. Food poisoning with vomiting for 12 hours in a middle-aged woman d. Hypoxemia for 4 days from pneumonia in an adult woman

d. Hypoxemia for 4 days from pneumonia in an adult woman

In the client with hypoventilation, which change in arterial blood gases does the nurse evaluate to determine whether treatment measures are being effective? a. Decreased arterial blood pH b. Decreased arterial blood carbon dioxide c. Increased arterial blood bicarbonate d. Increased arterial blood oxygen

d. Increased arterial blood oxygen

A nurse is teaching parents about diarrhea. Which statement by the parents indicates understanding of the teaching? a. Diarrhea results from a fluid deficit in the small intestine. b. Organisms destroy intestinal mucosal cells, resulting in an increased intestinal surface area. c. Malabsorption results in metabolic alkalosis. d. Increased motility results in impaired absorption of fluid and nutrients.

d. Increased motility results in impaired absorption of fluid and nutrients.

Which statement best describes why infants are at greater risk for dehydration than older children? a. Infants have an increased ability to concentrate urine. b. Infants have a greater volume of intracellular fluid. c. Infants have a smaller body surface area. d. Infants have an increased extracellular fluid volume.

d. Infants have an increased extracellular fluid volume.

In caring for a patient following lobectomy for lung cancer, which of the following should the nurse include in the plan of care? a. Position the patient on the operative side only. b. Avoid administering narcotic pain medications. c. Keep the patient on strict bed rest. d. Instruct the patient to cough and deep breathe.

d. Instruct the patient to cough and deep breathe.

Which is an initial priority intervention for a client with stress incontinence? a. Beginning medication and dietary teaching b. Referring the client to an incontinence clinic c. Assisting the client in finding absorbent pads d. Instructing the client to maintain an incontinence diary

d. Instructing the client to maintain an incontinence diary

A child with a history of fever of unknown origin, excessive bruising, lymphadenopathy, anemia, and fatigue is exhibiting symptoms most suggestive of a. Ewing sarcoma b. Wilms' tumor c. Neuroblastoma d. Leukemia

d. Leukemia

A client has been NPO after a colectomy with nasogastric (NG) suction in place. On assessment, the nurse finds the client reporting cramps in the calves. Which action by the nurse is most appropriate? a. Document findings and notify the physician. b. Stop suction and request that the laboratory draw arterial blood gases. c. Prepare to administer lorazepam (Ativan). d. Raise the siderails and notify the physician.

d. Raise the siderails and notify the physician.

When a client has an arterial blood pH of 7.48, which buffer action will bring the pH back to normal? a. Absorption of bicarbonate ions from the blood b. Release of bicarbonate ions into the blood c. Absorption of hydrogen ions from the blood d. Release of hydrogen ions into the blood

d. Release of hydrogen ions into the blood

Which interventions utilized by the nurse will indicate the ability to recognize a localized inflammatory response? a. Vigorous range-of-motion exercises b. Turn, cough, and deep breathe c. Orient to date, time, and place d. Rest, ice, and elevation

d. Rest, ice, and elevation

A client has severe metabolic alkalosis. Which nursing diagnosis does the nurse choose as the client's priority problem? a. Fluid volume excess related to reduced kidney function b. Fluid volume deficit related to increased insensitive fluid loss through lungs c. Risk for impaired skin integrity related to accompanying peripheral edema d. Risk for injury related to increased neuronal sensitivity from hypocalcemia

d. Risk for injury related to increased neuronal sensitivity from hypocalcemia

A client with chemotherapy-induced bone marrow suppression has received filgrastim (Neupogen). Which laboratory finding indicates that this therapy is effective for the client? a. Hematocrit is 28%. b. Hematocrit is 38%. c. Segmented neutrophil count is 2500/mm3. d. Segmented neutrophil count is 3500/mm3.

d. Segmented neutrophil count is 3500/mm3.

A pregnant woman at 37 weeks of gestation has had ruptured membranes for 26 hours. A cesarean section is performed for failure to progress. The fetal heart rate before birth is 180 beats/min with limited variability. At birth, the newborn has Apgar scores of 6 and 7 at 1 and 5 minutes and is noted to be pale and tachypneic. Based on the maternal history, the cause of this newborn's distress is most likely a. Hypoglycemia b. Phrenic nerve injury c. Respiratory distress syndrome d. Sepsis

d. Sepsis

A breast cancer survivor has chemotherapy-related cognitive impairment. Which area should the nurse assess? a. Pain b. Grief c. Nightmares d. Short-term memory

d. Short-term memory

A patient has an acute intravascular hemolytic reaction to a blood transfusion. After discontinuing the blood transfusion, which is the nurse's next action? a. Discontinue the IV catheter. b. Return the blood to the blood bank. c. Run normal saline through the existing tubing. d. Start normal saline at TKO rate using new tubing.

d. Start normal saline at TKO rate using new tubing.

The nurse correlates the role of suppressor genes in cancer development with which action? a. The presence of suppressor genes increases risks for gene damage by carcinogens. b. People with a greater number of suppressor genes are at increased risk for getting cancer. c. Suppressor genes enhance immune function, suppressing cancer development. d. Suppressor genes limit cell division, reducing risks for developing cancer.

d. Suppressor genes limit cell division, reducing risks for developing cancer.

A nurse is taking a history on a patient with cancer. Which assessment is priority? a. Fatigue b. Vision c. Dehydration d. Blood pressure

a. Fatigue

Which clinical finding is an overt sign of retinoblastoma in children? a. Whitish reflex in the eye b. Lymphadenopathy c. Bone pain d. Change in gait

a. Whitish reflex in the eye

The nurse is caring for a client who is intubated with an endotracheal tube and on a mechanical ventilator. The client is able to make sounds. What is the nurse's first action? a. Check cuff inflation on the endotracheal tube. b. Listen carefully to the client. c. Call the health care provider. d. Auscultate the lungs.

a. Check cuff inflation on the endotracheal tube.

What teaching is essential for a client who has received an injection of iodine-131? a. "Do not share a toilet with anyone else or let anyone clean your toilet." b. "You need to save all your urine for the next week." c. "No special precautions are needed because this type of radiation is weak." d. "Avoid all contact with other people until the radiation device is removed."

a. "Do not share a toilet with anyone else or let anyone clean your toilet."

A client is being admitted with a suspected diagnosis of bladder cancer. Which question assists the nurse in determining risk factors? a. "Do you smoke cigarettes?" b. "Do you use alcohol?" c. "Do you use recreational drugs?" d. "Do you take any prescription drugs?"

a. "Do you smoke cigarettes?"

The nurse and a new nurse in orientation are caring for a patient with pneumonia. Which statement by the new nurse will indicate a correct understanding of this condition? a. "An infectious disease like pneumonia may not pose a risk to others." b. "We need to isolate the patient in a private negative-pressure room." c. "Clinical signs and symptoms are not present in pneumonia." d. "The patient will not be able to return home."

a. "An infectious disease like pneumonia may not pose a risk to others."

The nurse is administering prescribed sodium nitroprusside (Nipride) intravenously to a client who has shock. Which nursing intervention is a priority when administering this medication? a. Ask if the client has chest pain every 30 minutes. b. Assess the client's blood pressure every 15 minutes. c. Monitor the client's urinary output every hour. d. Observe the client's extremities every 4 hours.

b. Assess the client's blood pressure every 15 minutes.

A nurse is preparing to start a blood transfusion. Which type of tubing will the nurse obtain? a. Two-way valves to allow the patient's blood to mix and warm the blood transfusing b. An injection port to mix additional electrolytes into the blood c. A filter to ensure that clots do not enter the patient d. An air vent to let bubbles into the blood

c. A filter to ensure that clots do not enter the patient

The nurse is caring for a group of patients. Which patient will the nurse see first? a. A patient with Clostridium difficile in droplet precautions b. A patient with tuberculosis in airborne precautions c. A patient with MRSA infection in contact precautions d. A patient with a lung transplant in protective environment precautions

a. A patient with Clostridium difficile in droplet precautions

A client states that his brain tumor is benign and does not need to be removed. What is the nurse's best response? a. "As your tumor grows, it can damage your brain, so it should be removed." b. "Benign tumors consist of normal cells, so removal is only for cosmetic purposes." c. "Benign tumors turn into cancer, so they should be removed as soon as possible." d. "Because benign tumors can migrate, they should be removed before they spread."

a. "As your tumor grows, it can damage your brain, so it should be removed."

What is the best response for the nurse to give a parent about contacting the physician regarding an infant with diarrhea? a. "Call your pediatrician if the infant has not had a wet diaper for 6 hours." b. "The pediatrician should be contacted if the infant has two loose stools in an 8-hour period." c. "Call the doctor immediately if the infant has a temperature greater than 100° F." d. "Notify the pediatrician if the infant naps more than 2 hours."

a. "Call your pediatrician if the infant has not had a wet diaper for 6 hours."

The nurse is assessing a new patient admitted to home health. Which questions will be most appropriate for the nurse to ask to determine the risk of infection? (Select all that apply.) a. "Can you explain the risk for infection in your home?" b. "Have you traveled outside of the United States?" c. "Will you demonstrate how to wash your hands?" d. "What are the signs and symptoms of infection?" e. "Are you able to walk to the mailbox?" f. "Who runs errands for you?"

a. "Can you explain the risk for infection in your home?" b. "Have you traveled outside of the United States?" c. "Will you demonstrate how to wash your hands?" d. "What are the signs and symptoms of infection?"

A client recovering from septic shock is preparing for discharge home. What priority information does the nurse include in the teaching plan for this client? a. "Clean your toothbrush with laundry bleach daily." b. "Bathe every other day with antimicrobial soap." c. "Wash your hands after changing pet litter boxes." d. "Use an electric razor when you shave your face."

a. "Clean your toothbrush with laundry bleach daily."

While collecting a health history on a patient admitted for colon cancer, which of the following questions would be a priority to ask this patient? a. "Have you noticed any blood in your stool?" b. "Have you been experiencing nausea?" c. "Do you have back pain?" d. "Have you noticed any swelling in your abdomen?"

a. "Have you noticed any blood in your stool?"

A client has had two episodes of bacterial cystitis in the last 6 months. Which questions should the nurse ask? (Select all that apply.) a. "How much water do you drink every day?" b. "Do you take estrogen replacement?" c. "Does anyone in your family have a history of cystitis?" d. "Do you have any condition that affects your immune system?" e. "Are you on steroids or other immune suppressant drugs?" f. "Do you drink grapefruit juice every day?"

a. "How much water do you drink every day?" b. "Do you take estrogen replacement?" d. "Do you have any condition that affects your immune system?" e. "Are you on steroids or other immune suppressant drugs?"

A client has a history of renal calculi. Which statement by the client indicates a good understanding of preventive measures? a. "I know I should drink at least 3 to 4 liters of fluid every day." b. "I can't eat much dairy or other sources of calcium." c. "Aspirin and aspirin-containing products can lead to stones." d. "The doctor will give me antibiotics at the first sign of a stone."

a. "I know I should drink at least 3 to 4 liters of fluid every day."

A client is being discharged and continues to be at risk for developing metabolic alkalosis. Which statement by the client indicates to the nurse that teaching has been effective? a. "I will avoid excess use of antacids." b. "I'll drink at least three glasses of milk daily." c. "I'll avoid medications containing aspirin." d. "I will not add salt to my food during meals."

a. "I will avoid excess use of antacids."

Which statement made by a client with stress incontinence indicates a need for clarification of nutrition therapy? a. "I will limit my total intake of fluids." b. "I will avoid drinking alcoholic beverages." c. "I will avoid drinking caffeinated beverages." d. "I will try to lose about 10% of my body weight."

a. "I will limit my total intake of fluids."

The nurse is caring for a patient who is undergoing chemotherapy and radiation for cancer. The patient asks the nurse about the value of cancer screening when therapy is over. What is the nurse's best response? a. "It should be done on an ongoing schedule." b. "It is not something that should be discussed right now." c. "It probably will not be needed since the cancer has been cured." d. "It usually is not done but can be done if the patient wants peace of mind."

a. "It should be done on an ongoing schedule."

The nurse is planning discharge education for a client who had an exploratory laparotomy. Which nursing statement is appropriate when teaching the client to monitor for early signs of shock? a. "Monitor how much urine you void and report a decrease in the amount." b. "Take your temperature daily and report any below-normal body temperatures." c. "Assess your radial pulse every day and report an irregular rhythm." d. "Monitor your bowel movements and report ongoing constipation or diarrhea."

a. "Monitor how much urine you void and report a decrease in the amount."

The intensive care nurse is educating the spouse of a client who is being treated for shock. The spouse states, "The doctor said she has shock. What is that?" What is the nurse's best response? a. "Shock occurs when oxygen to the body's tissues and organs is impaired." b. "Shock is a serious condition, but it is not a life-threatening emergency." c. "Shock progresses slowly and can be stopped by the body's normal compensation." d. "Shock is a condition that affects only specific body organs like the kidneys."

a. "Shock occurs when oxygen to the body's tissues and organs is impaired."

A young woman is being treated with amoxicillin (Amoxil) for a urinary tract infection. Which is the highest priority instruction for the nurse to give this client? a. "Use a second form of birth control while on the drug." b. "You will experience increased menstrual bleeding while on this drug." c. "You may experience an irregular heartbeat while on the drug." d. "Watch for blood in your urine while taking this drug."

a. "Use a second form of birth control while on the drug."

The nurse is teaching a client about self-catheterization in the home setting. Which instructions are applicable? (Select all that apply.) a. "Wash your hands before and after self-catheterization." b. "Use a large-lumen catheter for each catheterization." c. "Use lubricant on the tip of the catheter before insertion." d. "Self-catheterize every 12 hours." e. "Use sterile gloves for the procedure." f. "Maintain a specific schedule for catheterization."

a. "Wash your hands before and after self-catheterization." c. "Use lubricant on the tip of the catheter before insertion." f. "Maintain a specific schedule for catheterization."

The nurse is counseling a client who smokes and drinks heavily about cancer risk. The client is adamant that he or she will never stop smoking. Which question by the nurse is most appropriate? a. "Would you be willing to stop drinking alcohol?" b. "Have you ever tried the nicotine patch?" c. "Why are you so determined to continue smoking?" d. "Do you understand that smoking is the leading cause of cancer?"

a. "Would you be willing to stop drinking alcohol?"

The health care provider has ordered a hypotonic intravenous (IV) solution to be administered. Which IV bag will the nurse prepare? a. 0.45% sodium chloride (1/2 NS) b. 0.9% sodium chloride (NS) c. Lactated Ringer's (LR) d. Dextrose 5% in Lactated Ringer's (D5LR)

a. 0.45% sodium chloride (1/2 NS)

The nurse monitors the client with which condition most carefully for metabolic alkalosis? a. A critical illness receiving total parenteral nutrition b. Type 1 diabetes on once-daily insulin therapy c. Metastatic breast cancer on continuous IV morphine d. Asthma using an adrenergic agonist inhaler

a. A critical illness receiving total parenteral nutrition

A client has been placed on a ventilator. The physician has ordered that the ventilator be set to deliver a respiratory rate set of 28 breaths/min. The nurse questions the order, citing concerns about which acid-base problem? a. Acid deficit: alkalosis b. Base excess: alkalosis c. Acid excess: acidosis d. Base deficit: acidosis

a. Acid deficit: alkalosis

A nurse is caring for a diabetic patient with a bowel obstruction and has orders to ensure that the volume of intake matches the output. In the past 4 hours, the patient received dextrose 5% with 0.9% sodium chloride through a 22-gauge catheter infusing at 150 mL/hr and has eaten 200 mL of ice chips. The patient also has an NG suction tube set to low continuous suction that had 300-mL output. The patient has voided 400 mL of urine. After reporting these values to the health care provider, which order does the nurse anticipate? a. Add a potassium supplement to replace loss from output. b. Decrease the rate of intravenous fluids to 100 mL/hr. c. Administer a diuretic to prevent fluid volume excess. d. Discontinue the nasogastric suctioning.

a. Add a potassium supplement to replace loss from output.

A client is hospitalized for chemotherapy. The registered nurse intervenes when observing which action by the nursing assistant? a. Allowing the client to rest instead of making him or her perform oral hygiene b. Helping the client wash the groin and axillary areas every 12 hours c. Cutting food and opening food packages when the client's meal tray arrives d. Reminding the client to use the incentive spirometer every hour while awake

a. Allowing the client to rest instead of making him or her perform oral hygiene

Four patients arrive at the emergency department at the same time. Which patient will the nurse see first? a. An infant with temperature of 102.2° F and diarrhea for 3 days b. A teenager with a sprained ankle and excessive edema c. A middle-aged adult with abdominal pain who is moaning and holding her stomach d. An older adult with nausea and vomiting for 3 days with blood pressure 112/60

a. An infant with temperature of 102.2° F and diarrhea for 3 days

The nurse questions which activity for the client with thrombocytopenia? a. Application of warm compresses to bruises b. Cleaning teeth with a soft-bristled brush c. Taking acetaminophen (Tylenol) for pain d. Using stool softeners daily for constipation

a. Application of warm compresses to bruises

A client has just experienced a 90-second tonic-clonic seizure and has these arterial blood gas values: pH 6.88, HCO3- 22 mEq/L, PCO2 60 mm Hg, PO2 50 mm Hg. Which intervention by the nurse is most appropriate? a. Apply oxygen by mask or nasal cannula. b. Apply a paper bag over the client's nose and mouth. c. Administer 50 mL of sodium bicarbonate intravenously. d. Administer 50 mL of 20% glucose and 20 units of regular insulin.

a. Apply oxygen by mask or nasal cannula.

What is the best way for the nurse to communicate with a client who is intubated and is receiving mechanical ventilation? a. Ask the client to point to words on a board. b. Ask the client to blink for "yes" and "no." c. Have the client mouth words slowly. d. Teach the client some simple sign language.

a. Ask the client to point to words on a board.

The hand grasps of a client with acidosis have diminished since the previous assessment 1 hour ago. What action does the nurse take next? a. Assess client's rate, rhythm, and depth of respiration. b. Measure the client's pulse and blood pressure. c. Document findings and continue to monitor. d. Notify the physician as soon as possible.

a. Assess client's rate, rhythm, and depth of respiration.

The nurse is caring for a client on a ventilator when the high-pressure alarm sounds. What actions are most appropriate? (Select all that apply.) a. Assess the tubing for kinks. b. Assess whether the tubing has become disconnected. c. Determine the need for suctioning. d. Call the health care provider. e. Call the Rapid Response Team. f. Auscultate the client's lungs.

a. Assess the tubing for kinks. c. Determine the need for suctioning. f. Auscultate the client's lungs.

A client has the following arterial blood gases (ABGs): pH 7.30, HCO3- 22 mEq/L, PCO2 55 mm Hg, PO2 86 mm Hg. Which intervention by the nurse takes priority? a. Assessing the airway b. Administering bronchodilators c. Administering mucolytics d. Providing oxygen

a. Assessing the airway

A child with acute myeloblastic leukemia is scheduled to have a bone marrow transplant (BMT). The donor is the child's own umbilical cord blood that had been previously harvested and banked. This type of BMT is termed a. Autologous b. Allogeneic c. Syngeneic d. Stem cell

a. Autologous

The nurse is planning discharge teaching for a client who has acute myelogenous leukemia (AML). Which instruction does the nurse include in this client's discharge plan? a. Avoid contact sports. b. Refrain from intercourse. c. Apply heat to any bruised areas. d. Use aspirin for headaches.

a. Avoid contact sports.

The nurse is caring for a client who is receiving mechanical ventilation accompanied by positive end-expiratory pressure (PEEP). What assessment findings require immediate intervention? a. Blood pressure drop from 110/90 mm Hg to 80/50 mm/Hg b. Pulse oximetry value of 96% c. Arterial blood gas (ABG): pH, 7.40; PaO2, 80 mm Hg; PaCO2, 45 mm Hg; HCO3-, 26 mEq/L d. Urinary output of 30 mL/hr

a. Blood pressure drop from 110/90 mm Hg to 80/50 mm/Hg

The mother of a newborn asks the nurse what causes the baby to begin to breathe after delivery. What changes in the respiratory system stimulating respirations postnatally can the nurse explain to the mother? Select all that apply. a. Low oxygen levels in the infant's blood b. Rubbing the newborn with a towel or blanket c. Surfactant, a special lubricant in the lungs d. Increased blood flow to the infant's lungs e. Cold environment in the delivery room

a. Low oxygen levels in the infant's blood b. Rubbing the newborn with a towel or blanket e. Cold environment in the delivery room

In preparing a cancer risk reduction pamphlet for African-American clients, it is most important that the nurse include information on prevention and early detection for which types of cancer? a. Lung and prostate b. Bone and leukemia c. Skin and lymphoma d. Stomach and esophageal

a. Lung and prostate

A postoperative client received six units of packed red blood cells (PRBCs) for intraoperative blood loss. The nurse monitors the client for which acid-base imbalance? a. Metabolic alkalosis b. Metabolic acidosis c. Respiratory alkalosis d. Respiratory acidosis

a. Metabolic alkalosis

A client has colorectal cancer. Which activities are especially important for the nurse to conduct for this client? (Select all that apply.) a. Monitor liver function studies. b. Maintain accurate intake and output. c. Obtain daily weight using the same scale. d. Palpate lymph nodes at each clinic visit. e. Ask the client about changes in belly size.

a. Monitor liver function studies. d. Palpate lymph nodes at each clinic visit. e. Ask the client about changes in belly size.

It is determined that a client has a large pulmonary embolism (PE). Fibrinolytic therapy is initiated. What is the nurse's priority action? a. Monitor the client's oxygenation. b. Teach the client about potential side effects. c. Monitor the IV insertion site. d. Monitor for bleeding.

a. Monitor the client's oxygenation.

A client with chronic respiratory acidosis is receiving oxygen by nasal cannula at 6 L/min. The client's respiratory rate is 8 breaths/min. Which action by the nurse is the priority? a. Notify the Rapid Response Team and prepare for intubation. b. Change the nasal cannula to a mask and reassess in 10 minutes. c. Place the client in Fowler's position if he or she is able to tolerate it. d. Decrease the flow rate of oxygen to 2 to 4 L/min, and reassess.

a. Notify the Rapid Response Team and prepare for intubation.

A patient presents with pneumonia. Which priority intervention should be included in the plan of care for this patient? a. Observe the patient for decreased activity tolerance. b. Assume the patient is in pain and treat accordingly. c. Provide the patient ice chips as requested. d. Maintain the room temperature at 65° F.

a. Observe the patient for decreased activity tolerance.

A client has been admitted from a nursing home for a workup to determine the cause of several recent falls. What intervention by the nurse takes priority? a. Obtain a clean catch or catheterized urine specimen. b. Document the number of and causative factors for falls. c. Review the results of recent laboratory work for kidney function. d. Facilitate neurologic and social work consultations.

a. Obtain a clean catch or catheterized urine specimen.

A client who has septic shock is admitted to the hospital. What priority intervention does the nurse implement first? a. Obtain two sets of blood cultures. b. Administer the prescribed IV vancomycin (Vancocin). c. Obtain central venous pressure (CVP) measurements. d. Administer the prescribed IV norepinephrine (Levophed).

a. Obtain two sets of blood cultures.

The nurse wishes to present a cancer program to a group of people at high risk for cancer. In planning the program, which group does the nurse consider the priority? a. Older adults b. People who smoke c. Clients with family histories of cancer d. People with poor immune function

a. Older adults

Which client is at greatest risk for development of a bacterial cystitis? a. Older woman not taking estrogen replacement b. Older man with mild congestive heart failure c. Middle-aged woman who has never been pregnant d. Middle-aged man taking cyclophosphamide for cancer therapy

a. Older woman not taking estrogen replacement

The nurse is teaching about the process of passively moving water from an area of lower particle concentration to an area of higher particle concentration. Which process is the nurse describing? a. Osmosis b. Filtration c. Diffusion d. Active transport

a. Osmosis

A client admitted for difficulty breathing becomes worse. Which assessment findings indicate that the client has developed acute respiratory distress syndrome (ARDS)? (Select all that apply.) a. Oxygen administered at 100%, PaO2 60 b. Increased dyspnea c. Anxiety d. Chest pain e. Pitting pedal edema f. Clubbing of fingertips

a. Oxygen administered at 100%, PaO2 60 b. Increased dyspnea c. Anxiety

A client is hospitalized with urinary retention, has an indwelling catheter, and is getting IV fluids. Which intervention does the nurse add to the care plan to address the priority problem for this client? a. Perform catheter care per policy every shift. b. Encourage fluid intake to 1 liter/day. c. Apply a moisture barrier cream daily. d. Document accurate intake and output (I&O) each shift.

a. Perform catheter care per policy every shift.

While interviewing a 48-year-old patient during her annual physical examination, the nurse learns that she has never had a mammogram. The American Cancer Society recommends annual mammography screening starting at age 40. Before the nurse encourages this patient to begin annual screening, it is important for her to understand the reasons why women avoid testing. These reasons include (select all that apply) a. Reluctance to hear bad news b. Fear of x-ray exposure c. Belief that lack of family history makes this test unnecessary d. Expense of the procedure e. Having heard that the test is painful

a. Reluctance to hear bad news b. Fear of x-ray exposure d. Expense of the procedure e. Having heard that the test is painful

A patient is experiencing respiratory acidosis. Which organ system is responsible for compensation in this patient? a. Renal b. Endocrine c. Respiratory d. Gastrointestinal

a. Renal

A client has mild acidosis but after a day has not compensated for it. Which action by the nurse is best? a. Review the client's daily hemoglobin and hematocrit. b. Ask the laboratory to rerun today's arterial blood gases. c. Document the finding and notify the physician. d. Apply 2 L of oxygen via nasal cannula.

a. Review the client's daily hemoglobin and hematocrit.

Which diet would the nurse recommend to the mother of a child who is having mild diarrhea? a. Rice, potatoes, yogurt, cereal, and cooked carrots b. Bananas, rice, applesauce, and toast c. Apple juice, hamburger, and salad d. Whatever the child would like to eat

a. Rice, potatoes, yogurt, cereal, and cooked carrots

What is the priority problem for a client experiencing chemotherapy-induced anemia? a. Risk for injury related to fatigue b. Fatigue related to decreased oxygenation c. Body image problems related to skin color changes d. Inadequate nutrition related to anorexia

a. Risk for injury related to fatigue

The nurse is caring for multiple clients in the emergency department. The client with which condition is at highest risk for distributive shock? a. Severe head injury from a motor vehicle accident b. Diabetes insipidus from polycystic kidney disease c. Ischemic cardiomyopathy from severe coronary artery disease d. Vomiting of blood from a gastrointestinal ulcer

a. Severe head injury from a motor vehicle accident

The nurse is reviewing laboratory results. Which cation will the nurse observe is the most abundant in the blood? a. Sodium b. Chloride c. Potassium d. Magnesium

a. Sodium

A client admitted with respiratory difficulty and decreased oxygen saturation keeps pulling off the oxygen mask. What action does the nurse take? a. Stays with the client and replaces the oxygen mask b. Asks the client's spouse to hold the oxygen mask in place c. Restrains the client per facility policy d. Contacts the health care provider and requests sedation

a. Stays with the client and replaces the oxygen mask

The nurse observes a red butterfly-shaped rash that spreads across the child's cheeks and nose. This assessment finding is characteristic of which condition? a. Systemic lupus erythematosus (SLE) b. Rheumatic fever c. Kawasaki disease d. Anaphylactic reaction

a. Systemic lupus erythematosus (SLE)

What is the best response by the nurse to a parent asking about antidiarrheal medication for her 18-month-old child? a. "It is okay to give antidiarrheal medication to a young child as long as you follow the directions on the box for correct dosage." b. "Antidiarrheal medication is not recommended for young children because it slows the body's attempt to rid itself of the pathogen." c. "I'm sure your child won't like the taste, so give extra fluids when you give the medication." d. "Antidiarrheal medication will lessen the frequency of stools, but give your child Gatorade to maintain electrolyte balance."

b. "Antidiarrheal medication is not recommended for young children because it slows the body's attempt to rid itself of the pathogen."

In planning a teaching session for a client undergoing photodynamic therapy for lung cancer, the nurse includes which statements? (Select all that apply.) a. "This is a palliative treatment that should decrease your pain." b. "Avoid exposure to the sun for 1 to 3 months after the treatment." c. "Do not eat or drink anything before your treatments." d. "Do not remove skin markings between treatments." e. "You need to wear sunglasses to protect your eyes after treatments." f. "Make sure you keep your curtains closed at home afterward."

b. "Avoid exposure to the sun for 1 to 3 months after the treatment." e. "You need to wear sunglasses to protect your eyes after treatments." f. "Make sure you keep your curtains closed at home afterward."

An adult client who has a suspicious mammogram says that her mother died of bone cancer when she was around the same age. Which is the most important question for the nurse to ask this client? a. "Have any other members of your family had bone cancer?" b. "Did your mother ever have any other type of cancer?" c. "How old were you when you started your periods?" d. "Did your mother have regular mammograms?"

b. "Did your mother ever have any other type of cancer?"

Which client statement indicates a good understanding regarding antibiotic therapy for recurrent urinary tract infections? a. "If my urine becomes lighter and clearer, I can stop taking my medicine." b. "Even if I feel completely well, I should take the medication until it is gone." c. "When my urine no longer burns, I will no longer need to take the antibiotics." d. "If I have a fever higher than 100° F (37.8° C), I should take twice as much medicine."

b. "Even if I feel completely well, I should take the medication until it is gone."

Which statement by a school-age girl indicates the need for further teaching about the prevention of urinary tract infections (UTIs)? a. "I always wear cotton underwear." b. "I really enjoy taking a bubble bath." c. "I go to the bathroom every 3 to 4 hours." d. "I drink four to six glasses of fluid every day."

b. "I really enjoy taking a bubble bath."

Which statements said by patients indicate that the nurse's teaching regarding prevention of acid-base imbalances is successful? (Select all that apply.) a. "Baking soda is an effective inexpensive antacid." b. "I shall take my insulin on time every day." c. "My aspirin is on a high shelf away from children." d. "I have reliable transportation to dialysis sessions." e. "Fasting is a great way to lose weight rapidly."

b. "I shall take my insulin on time every day." c. "My aspirin is on a high shelf away from children." d. "I have reliable transportation to dialysis sessions."

Which statement by the patient indicates that she understands breast self-examination? a. "I will examine both breasts in two different positions." b. "I will perform breast self-examination 1 week after my menstrual period starts." c. "I will examine the outer upper area of the breast only." d. "I will use the palm of the hand to perform the examination."

b. "I will perform breast self-examination 1 week after my menstrual period starts."

A client says that she has heard that the origin of most cancers is genetic and wants "genetic testing because of a family history of cancer." What is the nurse's best response? a. "I will ask your physician about a referral for genetic testing." b. "Let's look at your family history back to your grandparents' generation." c. "Genetic testing is so expensive; let's talk about reducing your risk instead." d. "Inherited cancers are much more common in males than in females."

b. "Let's look at your family history back to your grandparents' generation."

A client is undergoing radiation therapy and says, "I will be so glad when this is over and I don't have to worry about my skin." What response by the nurse is most appropriate? a. "Unfortunately, your skin will be permanently damaged from the radiation." b. "You need to protect your skin from the sun for at least a year afterward." c. "You can get a prescription for special lotions that reduce the effects of radiation." d. "You're having skin problems? That is unusual; let me take a look at your skin."

b. "You need to protect your skin from the sun for at least a year afterward."

A nurse begins infusing a 250-mL bag of IV fluid at 1845 on Monday and programs the pump to infuse at 50 mL/hr. At what time should the infusion be completed? a. 2300 Monday b. 2345 Monday c. 0015 Tuesday d. 0045 Tuesday

b. 2345 Monday

Which personal factor places a client at risk for bladder cancer? a. Working in a lumber yard for 10 years b. 50-pack-year cigarette smoking history c. Numerous episodes of bacterial cystitis d. History of sexually transmitted diseases

b. 50-pack-year cigarette smoking history

In which patient will the nurse expect to see a positive Chvostek sign? a. A 7-year-old child admitted for severe burns b. A 24-year-old adult admitted for chronic alcohol abuse c. A 50-year-old patient admitted for an acute exacerbation of hyperparathyroidism d. A 75-year-old patient admitted for a broken hip related to osteoporosis

b. A 24-year-old adult admitted for chronic alcohol abuse

A nurse is caring for a patient whose ECG presents with changes characteristic of hypokalemia. Which assessment finding will the nurse expect? a. Dry mucous membranes b. Abdominal distention c. Distended neck veins d. Flushed skin

b. Abdominal distention

The nurse is assessing a client at risk for shock. The client's systolic blood pressure is 20 mm Hg lower than baseline. Which intervention does the nurse perform first? a. Increase the IV fluid rate. b. Administer oxygen. c. Notify the health care provider. d. Place the client in high Fowler's position.

b. Administer oxygen.

A hospitalized child has developed a methicillin-resistant Staphylococcus aureus (MRSA) infection. The nurse plans which interventions when caring for this child? Select all that apply. a. Airborne isolation b. Administration of vancomycin (Vancocin) c. Contact isolation d. Administration of mupirocin (Bactroban) ointment to the nares e. Administration of cefotaxime (Cefotetan)

b. Administration of vancomycin (Vancocin) c. Contact isolation d. Administration of mupirocin (Bactroban) ointment to the nares

The patient has contracted a urinary tract infection (UTI) while in the hospital. Which action will most likely increase the risk of a patient contracting a UTI? a. Reusing the patient's graduated receptacle to empty the drainage bag. b. Allowing the drainage bag port to touch the graduated receptacle. c. Emptying the urinary drainage bag at least once a shift. d. Irrigating the catheter infrequently.

b. Allowing the drainage bag port to touch the graduated receptacle.

In preparing a community teaching program, which information does the nurse plan to present to address secondary cancer prevention? a. Receiving cancer treatment with chemotherapy b. Annual measurement of prostate-specific antigen levels c. Avoiding known cancer-causing substances or conditions d. Having adolescent children receive the Gardasil vaccination

b. Annual measurement of prostate-specific antigen levels

The nurse correlates which condition with the following arterial blood gas values: pH 7.48, HCO3- 22 mEq/L, PCO2 28 mm Hg, PO2 98 mm Hg? a. Diarrhea and vomiting for 36 hours b. Anxiety-induced hyperventilation c. Chronic obstructive pulmonary disease d. Diabetic ketoacidosis and emphysema

b. Anxiety-induced hyperventilation

The nurse is caring for a patient with known coronary artery disease who has recently been diagnosed with lung cancer. What should the nurse do? a. Focus the assessment solely on the cancer diagnosis since it is the newer diagnosis. b. Ask questions about cardiac symptoms and their relationship to the cancer. c. Ignore symptom management and focus on palliative care. d. Say nothing because cancer survivors dislike prying.

b. Ask questions about cardiac symptoms and their relationship to the cancer.

A client has moderate metabolic alkalosis. What is the priority intervention for the nurse? a. Monitor daily laboratory values. b. Assess the client's muscle strength. c. Determine the cause of the problem. d. Teach the client preventive measures.

b. Assess the client's muscle strength.

A client has a fungal urinary tract infection. Which assessment by the nurse is most helpful? a. Palpating and percussing the kidneys and bladder b. Assessing medical history and current medical problems c. Performing a bladder scan to assess post-void residual d. Inquiring about recent travel to foreign countries

b. Assessing medical history and current medical problems

A child with a brain tumor is undergoing radiation therapy. What should the nurse include in the discharge instructions to the child's parents? Select all that apply. a. Apply over-the-counter creams to the area daily. b. Avoid excessive skin exposure to the sun. c. Use a washcloth when cleaning the area receiving radiation. d. Plan for adequate rest periods for the child. e. A darkening of the skin receiving radiation is expected.

b. Avoid excessive skin exposure to the sun. d. Plan for adequate rest periods for the child. e. A darkening of the skin receiving radiation is expected.

Which set of assessment data is consistent for a patient with severe infection that could lead to system failure? a. Blood pressure (BP) 92/52, pulse (P) 56 beats/min, respiratory rate (RR) 10 breaths/min, urine output 1200 mL in past 24 hours b. BP 90/48, P 112 beats/min, RR 26 breaths/min, urine output 240 mL in past 24 hours c. BP 112/64, P 98 beats/min, RR 18 breaths/min, urine output 2400 mL in past 24 hours d. BP 152/90, P 52 beats/min, RR 12 breaths/min, urine output 4800 mL in past 24 hours

b. BP 90/48, P 112 beats/min, RR 26 breaths/min, urine output 240 mL in past 24 hours

A client who was malnourished is being discharged. The nurse evaluates that teaching to decrease risk for the development of metabolic acidosis has been effective when the client states, "I will: a. Increase my milk intake to at least three glasses daily." b. Be sure to eat three well-balanced meals and a snack daily." c. Avoid taking pain medication and antihistamines together." d. Not add salt to food when cooking or during meals."

b. Be sure to eat three well-balanced meals and a snack daily."

The nurse understands that the type of precautions needed for children receiving chemotherapy is based on which actions of chemotherapeutic agents? a. Gastrointestinal upset b. Bone marrow suppression c. Decreased creatinine level d. Alopecia

b. Bone marrow suppression

The nurse receives the patient's most recent blood work results. Which laboratory value is of greatest concern? a. Sodium of 145 mEq/L b. Calcium of 15.5 mg/dL c. Potassium of 3.5 mEq/L d. Chloride of 100 mEq/L

b. Calcium of 15.5 mg/dL

The nurse is working in a long-term care facility where many clients use habit training to manage incontinence. Which action by unlicensed assistive personnel (UAP) requires intervention by the nurse? a. Toileting clients after meals b. Changing incontinence briefs when wet c. Encouraging clients to drink fluids d. Recording incontinence episodes

b. Changing incontinence briefs when wet

The infant with bronchopulmonary dysplasia (BPD) who has RSV bronchiolitis is a candidate for which treatment? a. Pancreatic enzymes b. Cool humidified oxygen c. Erythromycin intravenously d. Intermittent positive pressure ventilation

b. Cool humidified oxygen

A client with severe respiratory insufficiency becomes short of breath during activities of daily living. Which nursing intervention is best? a. Call the Rapid Response Team. b. Decrease involvement in care until the episode is past. c. Cluster morning activities to provide long rest periods. d. Space out interventions to provide for periods of rest.

b. Decrease involvement in care until the episode is past.

The nurse observes that the patient's calcium is elevated. When checking the phosphate level, what does the nurse expect to see? a. Increased b. Decreased c. Equal to calcium d. No change in phosphate

b. Decreased

A young child with HIV is receiving several antiretroviral drugs. The purpose of these drugs is to a. Cure the disease. b. Delay disease progression. c. Prevent the spread of disease. d. Treat Pneumocystis carinii pneumonia.

b. Delay disease progression.

What is an appropriate nursing action before surgery when caring for a child diagnosed with a Wilms' tumor? a. Limit fluid intake. b. Do not palpate the abdomen. c. Force oral fluids. d. Palpate the abdomen every 4 hours.

b. Do not palpate the abdomen.

What is an expected physical assessment finding for an adolescent with a diagnosis of Hodgkin disease? a. Protuberant, firm abdomen b. Enlarged, painless, firm cervical lymph nodes c. Soft tissue swelling d. Soft to hard, nontender mass in pelvic area

b. Enlarged, painless, firm cervical lymph nodes

The infection control nurse is reviewing data for the medical-surgical unit. The nurse notices an increase in postoperative infections from Aspergillus. Which type of health care-associated infection will the nurse report? a. Vector b. Exogenous c. Endogenous d. Suprainfection

b. Exogenous

A client presents with senile dementia, Alzheimer's type (SDAT), and incontinence. Which therapy will best help this client? a. Bladder training b. Habit training c. Exercise therapy d. Electrical stimulation

b. Habit training

A client is admitted to the emergency department several hours after a motor vehicle crash. The car's driver-side airbag was activated during the accident. Which assessment requires the nurse's immediate intervention? a. Disorientation b. Hemoptysis c. Pulse oximetry reading of 94% d. Chest pain with movement

b. Hemoptysis

Which symptoms in a client assist the nurse in confirming the diagnosis of pulmonary embolus (PE)? (Select all that apply.) a. Wheezes throughout lung fields b. Hemoptysis c. Sharp chest pain d. Flattened neck veins e. Hypotension f. Pitting edema

b. Hemoptysis c. Sharp chest pain e. Hypotension

The nurse is changing linens for a postoperative patient and feels a prick in the left hand. A nonactivated safe needle is noted in the linens. For which condition is the nurse most at risk? a. Diphtheria b. Hepatitis B c. Clostridium difficile d. Methicillin-resistant Staphylococcus aureus

b. Hepatitis B

A client is receiving treatment with levofloxacin (Levaquin). Which teaching topics does the nurse include in this client's care plan? a. How to assess blood pressure b. How to assess a radial pulse c. How to assess a carotid pulse d. How to assess respirations

b. How to assess a radial pulse

Which biologic characteristic is specific to normal differentiated adult cells but not to cancer cells? a. Anaplasia b. Hypertrophy c. Aneuploidy d. Loose adherence

b. Hypertrophy

The nurse is caring for a patient who has a bloodborne pathogen. The nurse splashes blood above the glove to intact skin while discontinuing an intravenous (IV) infusion. Which step(s) will the nurse take next? a. Obtain an alcohol swab, remove the blood with an alcohol swab, and continue care. b. Immediately wash the site with soap and running water, and seek guidance from the manager. c. Do nothing; accidentally getting splashed with blood happens frequently and is part of the job. d. Delay washing of the site until the nurse is finished providing care to the patient.

b. Immediately wash the site with soap and running water, and seek guidance from the manager.

While completing an assessment on a 6-month-old infant, which finding should the nurse recognize as a symptom of a brain tumor in an infant? a. Blurred vision b. Increased head circumference c. Vomiting when getting out of bed d. Headache

b. Increased head circumference

A client is in the emergency department after an overdose of an unknown substance. Which assessment findings does the nurse correlate with possible salicylate poisoning? a. Increased deep tendon reflexes b. Increased rate and depth of respiration c. Decreased capillary refill d. Decreased intestinal motility and paralytic ileus

b. Increased rate and depth of respiration

The nurse is assessing a client who has hypovolemic shock. Which laboratory value indicates that the client is at risk for acidosis? a. Decreased serum creatinine b. Increased serum lactic acid c. Increased urine specific gravity d. Decreased partial pressure of arterial carbon dioxide

b. Increased serum lactic acid

The pilot balloon on the endotracheal tube of a client being mechanically ventilated is deflated. What is the nurse's priority action? a. Nothing; this is required during ventilation. b. Inflate the cuff using minimal leak technique. c. Call the Rapid Response Team. d. Increase the tidal volume.

b. Inflate the cuff using minimal leak technique.

A child with non-Hodgkin lymphoma will be starting chemotherapy. What intervention is initiated before chemotherapy to prevent tumor lysis syndrome? a. Insertion of a central venous catheter b. Intravenous (IV) hydration containing sodium bicarbonate c. Placement of an externalized ventriculoperitoneal (VP) shunt d. Administration of pneumococcal and Haemophilus influenzae type B vaccines

b. Intravenous (IV) hydration containing sodium bicarbonate

A client has acute pancreatitis and a risk for acid-base imbalance. The nurse plans to assess for which manifestation consistent with this condition? a. Agitation b. Kussmaul respirations c. Seizures d. Positive Chvostek's sign

b. Kussmaul respirations

The patient has severe hyperthyroidism and will have surgery tomorrow. What assessment is most important for the nurse to assess in order to detect development of the acid-base imbalance for which the patient has highest risk? a. Urine output and color b. Level of consciousness c. Heart rate and blood pressure d. Lung sounds in lung bases

b. Level of consciousness

The nurse is caring for a patient in protective environment. Which actions will the nurse take? (Select all that apply.) a. Wear an N95 respirator when entering the patient's room. b. Maintain airflow rate greater than 12 air exchanges/hr. c. Place in special room with negative-pressure airflow. d. Open drapes during the daytime. e. Listen to the patient's interests. f. Place dried flowers in a plastic vase.

b. Maintain airflow rate greater than 12 air exchanges/hr. d. Open drapes during the daytime. e. Listen to the patient's interests.

A client with prostate cancer is taking estrogen daily to control tumor growth. He reports that his left calf is swollen and painful. Which is the nurse's best action? a. Instruct the client to keep the leg elevated. b. Measure and compare calf circumferences. c. Apply ice to the calf after massaging it. d. Document this expected response.

b. Measure and compare calf circumferences.

A client is receiving interleukin-2 (IL-2) for cancer. Which drug is the nurse prepared to administer if needed? a. Lorazepam (Ativan) b. Meperidine (Demerol) c. Furosemide (Lasix) d. Epoetin alfa (Epogen)

b. Meperidine (Demerol)

A patient is admitted for a bowel obstruction and has had a nasogastric tube set to low intermittent suction for the past 3 days. Which arterial blood gas values will the nurse expect to observe? a. Respiratory alkalosis b. Metabolic alkalosis c. Metabolic acidosis d. Respiratory acidosis

b. Metabolic alkalosis

The nurse is screening clients at a health fair. Which client is at highest risk for the development of colon cancer? a. Older white client with irritable bowel syndrome b. Middle-aged African-American client who smokes cigars c. Middle-aged Asian client who travels and eats out frequently d. Older American Indian client taking hormone replacement therapy

b. Middle-aged African-American client who smokes cigars

The nurse is caring for a child with acute respiratory distress syndrome (ARDS) associated with sepsis. Nursing actions should include: a. Forcing fluids b. Monitoring pulse oximetry c. Instituting seizure precautions d. Encouraging a high-protein diet

b. Monitoring pulse oximetry

The nurse is assessing the laboratory findings of a client with a urinary tract infection. The laboratory report notes a "shift to the left" in a client's white blood cell count. Which action by the nurse is most appropriate? a. Request that the laboratory perform a differential analysis on the white blood cells. b. Notify the health care provider and start an IV line for parenteral antibiotics. c. Instruct the client to begin straining all urine for renal calculi. d. Document the finding in the client's chart and continue to monitor.

b. Notify the health care provider and start an IV line for parenteral antibiotics.

Which clients are at highest risk for pulmonary embolism (PE)? (Select all that apply) a. Middle-aged client awaiting surgery b. Older adult with a 20-pack-year history of smoking c. Client who has been on bedrest for 3 weeks d. Obese client who has elevated platelets e. Middle-aged client with diabetes mellitus type 1 f. Older adult who has just had abdominal surgery

b. Older adult with a 20-pack-year history of smoking c. Client who has been on bedrest for 3 weeks d. Obese client who has elevated platelets f. Older adult who has just had abdominal surgery

The nurse is planning care for a client who has leukemia. Which intervention does the nurse include in the plan of care to prevent fatigue? a. Arrange for a family member to stay with the client. b. Plan care for times when the client has the most energy. c. Schedule for daily physicals and occupational therapy. d. Plan all activities to occur in the morning to allow for afternoon naps.

b. Plan care for times when the client has the most energy.

The nurse is caring for a client with a ventilation/perfusion mismatch who is receiving mechanical ventilation. Which intervention is a priority for this client? a. Administering antibiotics every 6 hours b. Positioning the client with the "good lung dependent" c. Making sure that the pilot balloon line on the endotracheal tube is deflated d. Ensuring that the client is able to speak clearly

b. Positioning the client with the "good lung dependent"

A nurse is assessing a patient. Which assessment finding should cause a nurse to further assess for extracellular fluid volume deficit? a. Moist mucous membranes b. Postural hypotension c. Supple skin turgor d. Pitting edema

b. Postural hypotension

A client has acidosis. Which laboratory finding is of greatest concern to the nurse? a. Sodium 154 mEq/L b. Potassium 5.9 mEq/L c. Calcium 8.9 mg/dL d. Magnesium 2.1 mg/dL

b. Potassium 5.9 mEq/L

A nurse is administering a diuretic to a patient and teaching the patient about foods to increase. Which food choices by the patient will best indicate successful teaching? a. Milk and cheese b. Potatoes and fresh fruit c. Canned soups and vegetables d. Whole grains and dark green leafy vegetables

b. Potatoes and fresh fruit

A nurse is working in a cancer facility that follows the Institute of Medicine's (IOM) recommendations for essential components of survivorship care. Which recommendations will be the nurse's focus? (Select all that apply.) a. Cessation of noncancer follow-up and care b. Prevention and detection of new and recurrent cancers c. Intervention for consequences of cancer and its treatment d. Coordination between specialists and primary care providers e. Surveillance for cancer spread, recurrence, or second cancers

b. Prevention and detection of new and recurrent cancers c. Intervention for consequences of cancer and its treatment d. Coordination between specialists and primary care providers e. Surveillance for cancer spread, recurrence, or second cancers

The nurse most likely would construct a three-generation pedigree for a client who had a relative treated for which cancer? a. Lung cancer b. Prostate cancer c. Cervical cancer d. Bone cancer

b. Prostate cancer

An occupational health nurse is working with management in a firm that provides commercial building restoration, including asbestos removal. Which action does the nurse recommend to management? a. Provide annual screening chest x-rays for those exposed to asbestos. b. Purchase protective gear and develop policies mandating its use. c. Offer "stop smoking" programs on site several times a year. d. Routinely distribute testing kits for occult fecal blood.

b. Purchase protective gear and develop policies mandating its use.

A 2-year-old child is brought into the emergency department after ingesting a medication that causes respiratory depression. For which acid-base imbalance will the nurse most closely monitor this child? a. Respiratory alkalosis b. Respiratory acidosis c. Metabolic acidosis d. Metabolic alkalosis

b. Respiratory acidosis

A client is being discharged from the emergency department with several broken ribs. For which acid-base imbalance does the nurse provide discharge teaching? a. Respiratory alkalosis from anxiety and hyperventilation b. Respiratory acidosis from inadequate ventilation c. Metabolic acidosis from calcium loss from broken bones d. Metabolic alkalosis from taking base-containing analgesics

b. Respiratory acidosis from inadequate ventilation

A client who has acidosis resulting from hypovolemic shock has been prescribed intravenous fluid replacement. Which fluid does the nurse prepare to administer? a. Normal saline b. Ringer's lactate c. 5% dextrose in water d. 5% dextrose in 0.45% normal saline

b. Ringer's lactate

What is a priority nursing diagnosis for the 4-year-old child newly diagnosed with leukemia? a. Ineffective Breathing Pattern related to mediastinal disease b. Risk for Infection related to immunosuppressed state c. Disturbed Body Image related to alopecia d. Impaired Skin Integrity related to radiation therapy

b. Risk for Infection related to immunosuppressed state

A client has small cell lung cancer. Which laboratory result requires immediate intervention by the nurse? a. Serum potassium of 5.1 mEq/L b. Serum sodium of 118 mEq/L c. Hematocrit of 45% d. Blood urea nitrogen (BUN) of 10 mg/dL

b. Serum sodium of 118 mEq/L

Which factor predisposes the urinary tract to infection? a. Increased fluid intake b. Short urethra in young girls c. Prostatic secretions in males d. Frequent emptying of the bladder

b. Short urethra in young girls

The nurse is providing an education session to an adult community group about the effects of smoking on infection. Which information is most important for the nurse to include in the educational session? a. Smoke from tobacco products clings to your clothing and hair. b. Smoking affects the cilia lining the upper airways in the lungs. c. Smoking can affect the color of the patient's fingernails. d. Smoking tobacco products can be very expensive.

b. Smoking affects the cilia lining the upper airways in the lungs.

A nurse is discontinuing a patient's peripheral IV access. Which actions should the nurse take? (Select all that apply.) a. Wear sterile gloves and a mask. b. Stop the infusion before removing the IV catheter. c. Use scissors to remove the IV site dressing and tape. d. Apply firm pressure with sterile gauze during removal. e. Keep the catheter parallel to the skin while removing it. f. Apply pressure to the site for 2 to 3 minutes after removal.

b. Stop the infusion before removing the IV catheter. e. Keep the catheter parallel to the skin while removing it. f. Apply pressure to the site for 2 to 3 minutes after removal.

Which statements about urge incontinence and stress incontinence are true? (Select all that apply.) a. Urge incontinence involves a post-voiding residual volume less than 50 mL. b. Stress incontinence occurs because of weak pelvic floor muscles. c. Stress incontinence usually occurs in people with dementia. d. Urge incontinence can be managed by increasing fluid intake. e. Urge incontinence occurs because of abnormal bladder contractions.

b. Stress incontinence occurs because of weak pelvic floor muscles. e. Urge incontinence occurs because of abnormal bladder contractions.

The nurse is preparing to administer sodium nitroprusside (Nipride) to a client. Which important action related to the administration of this drug does the nurse implement? a. Assess the client's respiratory rate. b. Administer the medication with gravity tubing. c. Protect the medication from light with an opaque bag. d. Monitor for hypertensive crisis.

c. Protect the medication from light with an opaque bag.

A cancer survivor patient has anxiety and depression. Which therapies should the nurse include in the plan of care? (Select all that apply.) a. Keep information to a minimum with health care providers. b. Teach the use of problem-oriented coping processes. c. Encourage the use of social support systems. d. Use cognitive behavioral interventions. e. Schedule exercise when convenient.

b. Teach the use of problem-oriented coping processes. c. Encourage the use of social support systems. d. Use cognitive behavioral interventions.

A nurse is administering a blood transfusion. Which assessment finding will the nurse report immediately? a. Blood pressure 110/60 b. Temperature 101.3° F c. Poor skin turgor and pallor d. Heart rate of 100 beats/min

b. Temperature 101.3° F

What is the most important factor in determining the rate of fluid replacement in the dehydrated child? a. The child's weight b. The type of dehydration c. Urine output d. Serum potassium level

b. The type of dehydration

The patient is hyperventilating from anxiety and abdominal pain. Which assessment findings should the nurse attribute to respiratory alkalosis? (Select all that apply.) a. Skin pale and cold b. Tingling of fingertips c. Heart rate of 102 d. Numbness around mouth e. Cramping in feet

b. Tingling of fingertips d. Numbness around mouth e. Cramping in feet

The circulating nurse in the operating room is observing the surgical technologist while applying a sterile gown and gloves to care for a patient having an appendectomy. Which behaviors indicate to the nurse that the procedure by the surgical technologist is correct? (Select all that apply.) a. Ties the back of own gown b. Touches only the inside of gown c. Slips arms into arm holes simultaneously d. Extended fingers fully into both of the gloves e. Uses hands covered by sleeves to open gloves f. Applies surgical cap and face mask in the operating suite

b. Touches only the inside of gown c. Slips arms into arm holes simultaneously d. Extended fingers fully into both of the gloves e. Uses hands covered by sleeves to open gloves

A client has the following arterial blood results: pH 7.12, HCO3- 22 mEq/L, PCO2 65 mm Hg, PO2 56 mm Hg. The nurse correlates these values with which clinical situation? a. Diabetic ketoacidosis in a person with emphysema b. Tracheal obstruction related to aspiration of a hot dog c. Anxiety-induced hyperventilation in an adolescent d. Diarrhea for 36 hours in an older, frail woman

b. Tracheal obstruction related to aspiration of a hot dog

The nurse is caring for a client with a pulmonary embolus who also has right-sided heart failure. Which symptom will the nurse need to intervene for immediately? a. Respiratory rate of 28 breaths/min b. Urinary output of 10 mL/hr c. Heart rate of 100 beats/min d. Dry cough

b. Urinary output of 10 mL/hr

The nurse is caring for a patient with a diagnosed case of Clostridium difficile. The nurse expects to implement which of the following interventions? (Select all that apply.) a. Administration of protease inhibitors b. Use of personal protective equipment c. Patient teaching on methods to inhibit transmission d. Preventing visitors from entering the room e. Administration of intravenous fluids f. Strict monitoring of intake and output

b. Use of personal protective equipment c. Patient teaching on methods to inhibit transmission e. Administration of intravenous fluids f. Strict monitoring of intake and output

A client's radiation implant has become dislodged overnight, and the nurse finds it in the client's bed. What does the nurse do first? a. Assess the client's skin for radiation burns. b. Use tongs to put the implant into the radiation container. c. Notify the safety officer and move the client to a different room. d. Don gloves and attempt to replace the implant.

b. Use tongs to put the implant into the radiation container.

Which nursing action will most likely increase a patient's risk for developing a health care-associated infection? a. Uses surgical aseptic technique to suction an airway b. Uses a clean technique for inserting a urinary catheter c. Uses a cleaning stroke from the urinary meatus toward the rectum d. Uses a sterile bottled solution more than once within a 24-hour period

b. Uses a clean technique for inserting a urinary catheter

The nurse would incorporate which of the following into the plan of care as a primary prevention strategy for reduction of the risk for cancer? a. Yearly mammography for women aged 40 years and older b. Using skin protection during sun exposure while at the beach c. Colonoscopy at age 50 and every 10 years as follow-up d. Yearly prostate specific antigen (PSA) and digital rectal exam for men aged 50 and over

b. Using skin protection during sun exposure while at the beach

The patient experienced a surgical procedure, and Betadine was utilized as the surgical prep. Two days postoperatively, the nurse's assessment indicates that the incision is red and has a small amount of purulent drainage. The patient reports tenderness at the incision site. The patient's temperature is 100.5° F, and the WBC is 10,500/mm3. Which action should the nurse take first? a. Plan to change the surgical dressing during the shift. b. Utilize SBAR to notify the primary health care provider. c. Reevaluate the temperature and white blood cell count in 4 hours. d. Check to see what solution was used for skin preparation in surgery.

b. Utilize SBAR to notify the primary health care provider.

The nurse is preparing to insert a urinary catheter. The nurse is using open gloving to apply the sterile gloves. Which steps will the nurse take? (Select all that apply.) a. While putting on the first glove, touch only the outside surface of the glove. b. With gloved dominant hand, slip fingers underneath second glove cuff. c. Remove outer glove package by tearing the package open. d. Lay glove package on clean flat surface above waistline. e. Glove the dominant hand of the nurse first. f. After second glove is on, interlock hands.

b. With gloved dominant hand, slip fingers underneath second glove cuff. d. Lay glove package on clean flat surface above waistline. e. Glove the dominant hand of the nurse first. f. After second glove is on, interlock hands.

Which client does the nurse assess for potential metabolic acidosis? a. Client admitted after collapsing during a marathon run b. Young adult following a carbohydrate-free diet c. Older adult with asthma who is on long-term steroid therapy d. Older client on antacids for gastroesophageal reflux disease

b. Young adult following a carbohydrate-free diet

The nurse prepares to administer bicarbonate intravenously to the client with which clinical manifestations? a. pH 7.28, HCO3- 22 mEq/L, PCO2 52 mm Hg, PO2 82 mm Hg secondary to an acute asthma attack b. pH 7.28, HCO3- 16 mEq/L, PCO2 45 mm Hg, PO2 98 mm Hg secondary to excessive diarrhea c. Client with chronic emphysema and bronchitis who has the following arterial blood gases: pH 7.30, HCO3- 30 mEq/L, PCO2 60 mm Hg, PO2 72 mm Hg secondary to chronic bronchitis and emphysema d. pH 7.31, HCO3- 20 mEq/L, PCO2 34 mm Hg, PO2 96 mm Hg secondary to a urinary tract infection and type 2 diabetes

b. pH 7.28, HCO3- 16 mEq/L, PCO2 45 mm Hg, PO2 98 mm Hg secondary to excessive diarrhea

Which blood gas result will the nurse expect to observe in a patient with respiratory alkalosis? a. pH 7.60, PaCO2 40 mm Hg, HCO3- 30 mEq/L b. pH 7.53, PaCO2 30 mm Hg, HCO3- 24 mEq/L c. pH 7.35, PaCO2 35 mm Hg, HCO3- 26 mEq/L d. pH 7.25, PaCO2 48 mm Hg, HCO3- 23 mEq/L

b. pH 7.53, PaCO2 30 mm Hg, HCO3- 24 mEq/L

A client with severe bacterial cystitis is prescribed cefadroxil (Duricef) and phenazopyridine (Pyridium). What statement by the client indicates an accurate understanding of these medications? a. "I will not take these drugs with food or milk." b. "I will stop these drugs if I think I am pregnant." c. "An orange color in my urine won't alarm me." d. "I will try to drink a liter of cranberry juice daily."

c. "An orange color in my urine won't alarm me."

A nurse is reviewing the white blood cell count with differential for a client receiving chemotherapy for cancer. Which finding alerts the nurse to the possibility of sepsis? a. Total white blood cell count is 9000/mm3. b. Lymphocytes outnumber basophils. c. "Bands" outnumber "segs." d. Monocyte count is 1800/mm3.

c. "Bands" outnumber "segs."

A confused client is hospitalized for possible pneumonia and is admitted from the emergency department with an indwelling catheter in place. During interdisciplinary rounds the following day, what question by the nurse takes priority? a. "Do you want daily weights on this client?" b. "Will the client be able to return home?" c. "Can we discontinue the in-dwelling catheter?" d. "Should we get another chest x-ray today?"

c. "Can we discontinue the in-dwelling catheter?"

Which statement by a mother about antiretroviral agents for the management for her 5-year-old child with acquired immunodeficiency syndrome (AIDS) indicates that she has a good understanding? a. "When my child's pain increases, I double the recommended dosage of antiretroviral medication." b. "Addiction is a risk, so I only use the medication as ordered." c. "Doses of the antiretroviral medication are selected on the basis of my child's age and growth." d. "By the time my child is an adolescent she will not need her antiretroviral medications any longer."

c. "Doses of the antiretroviral medication are selected on the basis of my child's age and growth."

A client who has just had a mastectomy is crying. When the nurse asks about her crying, the client responds, "I know I shouldn't cry because this surgery may well save my life." What is the nurse's best response? a. "It is all right to cry. Mourning this loss will help make you stronger." b. "I know this is hard, but your chances of survival are better now." c. "I can arrange for someone who had a mastectomy to come visit if you like." d. "How have you coped with difficult situations in the past?"

c. "I can arrange for someone who had a mastectomy to come visit if you like."

Which statement indicates that the client needs more teaching about mucositis? a. "I will rinse my mouth with water after every meal." b. "I will use a soft-bristled toothbrush to prevent trauma." c. "I should use an alcohol-based mouth rinse to kill bacteria." d. "I cannot use floss because it may irritate my gums."

c. "I should use an alcohol-based mouth rinse to kill bacteria."

What is the nurse's best response to a mother whose child has a diagnosis of acute lymphoblastic leukemia and is expressing guilt about not having responded sooner to her boy's symptoms? a. "You should always call the physician when your child has a change in what is normal for him." b. "It is better to be safe than sorry." c. "It is not uncommon for parents not to notice subtle changes in their children's health." d. "I hope this delay does not affect the treatment plan."

c. "It is not uncommon for parents not to notice subtle changes in their children's health."

A client is receiving acetohydroxamic acid (Lithostat). Which statement by the client indicates a good understanding of this therapy? a. "I should finish this antibiotic even if I am feeling better." b. "I need to drink a full glass of water when I take this drug." c. "My blood will be drawn occasionally for kidney function tests." d. "This medication may turn my urine bright orange and stain my clothes."

c. "My blood will be drawn occasionally for kidney function tests."

A client has known lung cancer and has been admitted for abdominal pain and jaundice. A computed tomography (CT) scan reveals tumors in the client's liver. The client is distraught and says, "So now I have liver cancer too?" Which response by the nurse is most appropriate? a. "Yes, liver cancer is common in people who already have lung cancer." b. "Yes, your chemotherapy left you vulnerable to a virus that causes liver cancer." c. "No, the tumors are actually from your lung cancer, which has metastasized." d. "No, having tumors in two different organs is rare; you probably have hepatitis."

c. "No, the tumors are actually from your lung cancer, which has metastasized."

A client scheduled to undergo radiation therapy for breast cancer asks why 6 weeks of daily treatment is necessary. What is the nurse's best response? a. "Your cancer is widespread and requires more than the usual amount of radiation treatment." b. "Giving larger doses of radiation for a shorter period of time does not produce better effects and has worse side effects." c. "Research has shown that more cancer cells are killed if radiation is given in smaller doses over a longer time period." d. "It is less likely that your hair will fall out or that you will become anemic if radiation is given in this manner."

c. "Research has shown that more cancer cells are killed if radiation is given in smaller doses over a longer time period."

The middle-aged client with lung cancer asks whether his adult children are at increased risk for this cancer. What is the nurse's best response? a. "This disease is a random event and there is no way to prevent it." b. "This disease is inherited, so your children have a 50% risk for developing it." c. "Smoking is the main cause. Helping your children not smoke decreases their risk." d. "They can avoid cancer by decreasing the fat they eat and by exercising more."

c. "Smoking is the main cause. Helping your children not smoke decreases their risk."

The student nurse overhears several staff members referring to a client who is receiving chemotherapy as having "chemo brain." The student asks the instructor what that means. Which response by the instructor is best? a. "That is an awful thing to say and the staff should not call a client by that name." b. "It refers to the client's brain as being irreversibly damaged by the chemotherapy." c. "The client has reduced cognitive function that may last for several years." d. "The client has delirium related to the toxic effects of the chemotherapy."

c. "The client has reduced cognitive function that may last for several years."

A client is admitted with mixed respiratory and metabolic acidosis secondary to bronchitis and diabetic ketoacidosis. The nurse evaluates that teaching about the client's confusion was effective when a family member makes which statement? a. "It is too early to tell if the ketoacidosis will cause permanent changes." b. "Her memory will improve, but loss of some brain cells has occurred." c. "The confusion should clear when oxygen and electrolyte levels are normal." d. "The confusion should clear when blood glucose levels and other laboratory tests are normal."

c. "The confusion should clear when oxygen and electrolyte levels are normal."

A client with bladder cancer is scheduled to have intravesical chemotherapy. Which statement made by the client indicates correct understanding of this therapy? a. "My hair will start growing back in 3 to 6 weeks after chemotherapy is over." b. "My white blood cell count will drop and I will be at increased risk for infection." c. "This type of chemotherapy is used when no distant metastases are present." d. "Chemotherapy only controls cancer, so I will also need radiation."

c. "This type of chemotherapy is used when no distant metastases are present."

A client is on chemotherapy and has a platelet count of 25,000. Which intervention is most important to teach this client? a. "Eat a low-bacteria diet." b. "Take your temperature daily." c. "Use a soft-bristled toothbrush." d. "Avoid alcohol-based mouthwashes."

c. "Use a soft-bristled toothbrush."

A client brought to the emergency department after a motor vehicle accident is suspected of having internal bleeding. Which question does the nurse ask to determine whether the client is in the early stages of hypovolemic shock? a. "Are you more thirsty than normal?" b. "When was the last time you urinated?" c. "What is your normal heart rate?" d. "Is your skin usually cool and pale?"

c. "What is your normal heart rate?"

A female adult patient presents to the clinic with reports of a white discharge and itching in the vaginal area. A nurse is taking a health history. Which question is the priority? a. "When was the last time you visited your primary health care provider?" b. "Has this condition affected your eating habits in any way?" c. "What medications are you currently taking?" d. "Are you able to sleep at night?"

c. "What medications are you currently taking?"

The nurse is caring for a group of medical-surgical patients. Which patient is most at risk for developing an infection? a. A patient who is in observation for chest pain b. A patient who has been admitted with dehydration c. A patient who is recovering from a right total hip surgery d. A patient who has been admitted for stabilization of heart problems

c. A patient who is recovering from a right total hip surgery

What is caused by a virus that primarily infects a specific subset of T lymphocytes, the CD4+ T cells? a. Wiskott-Aldrich syndrome b. Idiopathic thrombocytopenic purpura c. Acquired immunodeficiency syndrome (AIDS) d. Severe combined immunodeficiency disease

c. Acquired immunodeficiency syndrome (AIDS)

The nurse is caring for a client who has hypovolemic shock. After administering oxygen, what is the priority intervention for this client? a. Administer an aminoglycoside. b. Initiate a dopamine hydrochloride (Intropin) drip. c. Administer crystalloid fluids. d. Initiate an intravenous heparin drip.

c. Administer crystalloid fluids.

The nurse assesses a client admitted for chest trauma who reports dyspnea. The nurse finds tracheal deviation and a pulse oximetry reading of 86%. What is the nurse's priority intervention? a. Notify the health care provider and document the symptoms. b. Intubate the client and prepare for mechanical ventilation. c. Administer oxygen and prepare for chest tube insertion. d. Administer an intermittent positive-pressure breathing treatment.

c. Administer oxygen and prepare for chest tube insertion.

In a client 4 minutes post cardiac arrest, the nurse correlates the largest source of excess hydrogen ions with which cause? a. Excess renal retention of carbon dioxide due to hypoxia b. Release of intracellular acids due to widespread tissue destruction c. Anaerobic metabolism, leading to the buildup of lactic acid d. Using fat as a fuel source, resulting in increased fat degradation

c. Anaerobic metabolism, leading to the buildup of lactic acid

The nurse is caring for a client with acute respiratory distress syndrome (ARDS) who is receiving mechanical ventilation and positive end-expiratory pressure (PEEP). The alarm sounds, indicating decreased pressure in the system. What is the nurse's best action? a. Change the client's position. b. Suction the client. c. Assess lung sounds. d. Turn off the pressure alarm.

c. Assess lung sounds.

The nurse auscultates the lungs of a client on mechanical ventilation and hears vesicular breath sounds throughout the right side but decreased sounds on the left side of the chest. What is the nurse's best action? a. Turn the client to the right side. b. Elevate the head of the bed. c. Assess placement of the endotracheal (ET) tube. d. Suction the client.

c. Assess placement of the endotracheal (ET) tube.

The pressure reading during inspiration on the ventilator of a client receiving mechanical ventilation is fluctuating widely. What is the nurse's first action? a. Determine whether an air leak is present in the client's endotracheal tube cuff. b. Have the respiratory therapist check the pressure settings. c. Assess the client's oxygenation. d. Manually ventilate the client with a resuscitation bag.

c. Assess the client's oxygenation.

It is most important that the nurse include which activity for the young adult client with Down syndrome? a. Encouraging more fruit and leafy green vegetables in the diet b. Teaching him how to perform testicular self-examination c. Assessing the skin for unusual bruises and petechiae d. Testing the client's stool for occult blood

c. Assessing the skin for unusual bruises and petechiae

A client has the following arterial blood gases: pH 7.30, HCO3- 17 mEq/L, PCO2 25 mm Hg, PO2 98 mm Hg. Which intervention by the nurse is most appropriate? a. Prepare to give intravenous sodium bicarbonate. b. Document the findings and continue to assess. c. Assist the physician in determining the cause. d. Administer oxygen at 2 L per nasal cannula.

c. Assist the physician in determining the cause.

A nurse determines that parents understood the teaching from the pediatric oncologist if the parents indicate that which test confirms the diagnosis of leukemia in children? a. Complete blood cell count (CBC) b. Lumbar puncture c. Bone marrow biopsy d. Computed tomography (CT) scan

c. Bone marrow biopsy

A client has a family history of colon cancer. Which laboratory tests are ordered to rule out colon cancer? a. Cholesterol b. Serum lipase c. Carcinoembryonic antigen d. Xylose absorption

c. Carcinoembryonic antigen

The nurse is caring for a patient in labor and delivery. When near completing an assessment of the patient's cervix, the electronic infusion device being used on the intravenous (IV) infusion alarms. Which sequence of actions is most appropriate for the nurse to take? a. Complete the assessment, remove gloves, and silence the alarm. b. Discontinue the assessment, silence the alarm, and assess the intravenous site. c. Complete the assessment, remove gloves, wash hands, and assess the intravenous infusion. d. Discontinue the assessment, remove gloves, use hand gel, and assess the intravenous infusion.

c. Complete the assessment, remove gloves, wash hands, and assess the intravenous infusion.

Which diagnostic test is used to confirm a suspected diagnosis of breast cancer? a. Mammogram b. Ultrasound c. Core needle biopsy d. CA 15-3

c. Core needle biopsy

A nurse manager on an oncology nursing unit notes an increased incidence of infection and serious consequences for clients on the unit. Which action by the nursing manager is most beneficial in this situation? a. Review asepsis policies at a mandatory in-service for staff. b. Spot-check all staff for good handwashing practices. c. Develop standard protocols to identify and treat clients with infection. d. Institute protective precautions for all clients receiving chemotherapy.

c. Develop standard protocols to identify and treat clients with infection.

A client is receiving a chemotherapeutic agent intravenously through a peripheral line. What is the nurse's first action when the client reports burning at the site? a. Check for a blood return. b. Slow the rate of infusion. c. Discontinue the infusion. d. Apply a cold compress.

c. Discontinue the infusion.

While the nurse is taking a patient history, the nurse discovers the patient has a type of diabetes that results from a head injury and does not require insulin. Which dietary change should the nurse share with the patient? a. Reduce the quantity of carbohydrates ingested to lower blood sugar. b. Include a serving of dairy in each meal to elevate calcium levels. c. Drink plenty of fluids throughout the day to stay hydrated. d. Avoid food high in acid to avoid metabolic acidosis.

c. Drink plenty of fluids throughout the day to stay hydrated.

What is the drug of choice the nurse should administer in the acute treatment of anaphylaxis? a. Diphenhydramine b. Histamine inhibitor (cimetidine) c. Epinephrine d. Albuterol

c. Epinephrine

The nurse monitors for which acid-base imbalance in a client who has hypoxemia? a. Reduced carbon dioxide production leading to alkalosis b. Reduced carbon dioxide retention leading to alkalosis c. Excess carbon dioxide production leading to acidosis d. Excess carbon dioxide retention leading to acidosis

c. Excess carbon dioxide production leading to acidosis

A client is hospitalized with a urinary tract infection (UTI). Which clinical manifestation alerts the nurse to the possibility of a complication from the UTI? a. Burning on urination b. Cloudy, dark urine c. Fever and chills d. Hematuria

c. Fever and chills

Nursing measures that help prevent postpartum urinary tract infection include a. Promoting bed rest for 12 hours after delivery b. Discouraging voiding until the sensation of a full bladder is present c. Forcing fluids to at least 3000 mL/day d. Encouraging the intake of orange, grapefruit, or apple juice

c. Forcing fluids to at least 3000 mL/day

Children receiving long-term systemic corticosteroid therapy are most at risk for a. Hypotension b. Dilation of blood vessels in the cheeks c. Growth delays d. Decreased appetite and weight loss

c. Growth delays

Which assessment is most relevant to the care of an infant with dehydration? a. Temperature, heart rate, and blood pressure. b. Respiratory rate, oxygen saturation, and lung sounds. c. Heart rate, sensorium, and skin color. d. Diet tolerance, bowel function, and abdominal girth.

c. Heart rate, sensorium, and skin color.

The nurse observes edema in a patient who has venous congestion from right heart failure. Which type of pressure facilitated the formation of the patient's edema? a. Osmotic b. Oncotic c. Hydrostatic d. Concentration

c. Hydrostatic

The nurse is monitoring a client in hypovolemic shock who has been placed on a dopamine hydrochloride (Intropin) drip. Which manifestation is a desired response to this medication? a. Decrease in blood pressure b. Increase in heart rate c. Increase in cardiac output d. Decrease in mean arterial pressure

c. Increase in cardiac output

A client was admitted 2 days ago with early stages of septic shock. Today the nurse notes that the client's systolic blood pressure, pulse pressure, and cardiac output are decreasing rapidly. Which intervention does the nurse do first? a. Insert a Foley catheter to monitor urine output closely. b. Ask the client's family to come to the hospital because death is near. c. Initiate the prescribed dobutamine (Dobutrex) intravenous drip. d. Obtain blood cultures before administering the next dose of antibiotics.

c. Initiate the prescribed dobutamine (Dobutrex) intravenous drip.

What should the nurse teach parents about oral hygiene for the child receiving chemotherapy? a. Brush the teeth briskly to remove bacteria. b. Use a mouthwash that contains alcohol. c. Inspect the child's mouth daily for ulcers. d. Perform oral hygiene twice a day.

c. Inspect the child's mouth daily for ulcers.

A client has metabolic alkalosis. Which laboratory results is the nurse most likely to assess as consistent with this condition? a. Na+ 134 mg/dL b. Mg2+ 1.5 mg/dL c. K+ 3.1 mEq/L d. Ca2+ 11.5 mg/dL

c. K+ 3.1 mEq/L

Children with non-Hodgkin lymphoma are at risk for complications resulting from tumor lysis syndrome (TLS). The nurse should assess for a. Liver failure b. CNS deficit c. Kidney failure d. Respiratory distress

c. Kidney failure

The nurse providing education regarding breast care should explain to the woman that fibrocystic changes in breasts are a. A disease of the milk ducts and glands in the breasts b. A pre-malignant disorder characterized by lumps found in the breast tissue c. Lumpiness with pain and tenderness found in varying degrees in the breast tissue of healthy women during menstrual cycles d. Lumpiness accompanied by tenderness after menses

c. Lumpiness with pain and tenderness found in varying degrees in the breast tissue of healthy women during menstrual cycles

The nurse is caring for a patient with leukemia and is preparing to provide fluids through a vascular access (IV) device. Which nursing intervention is a priority in this procedure? a. Review the procedure with the patient. b. Position the patient comfortably. c. Maintain surgical aseptic technique. d. Gather available supplies.

c. Maintain surgical aseptic technique.

A client has a prolonged fever. For which acid-base imbalance does the nurse assess the client further? a. Metabolic acidosis from excess bicarbonate production b. Metabolic alkalosis from dehydration and hyperkalemia c. Metabolic acidosis from increased production of hydrogen ions d. Respiratory alkalosis from impaired gas exchange

c. Metabolic acidosis from increased production of hydrogen ions

The nurse is aware that an abdominal mass found in a 10-month-old infant corresponds with which childhood cancer? a. Osteogenic sarcoma b. Rhabdomyosarcoma c. Neuroblastoma d. Non-Hodgkin lymphoma

c. Neuroblastoma

The nurse assesses a client who suffered chest trauma and finds that the left chest sucks in during inhalation and out during exhalation. The client's oxygen saturation has dropped from 94% to 86%. What is the priority action by the nurse? a. Encourage the client to take deep, controlled breaths. b. Document findings and continue to monitor the client. c. Notify the health care provider and prepare for intubation. d. Stabilize the chest wall with rib binders.

c. Notify the health care provider and prepare for intubation.

The nurse is caring for several clients on the respiratory floor. Which client does the nurse assess most carefully for the development of acute respiratory distress syndrome (ARDS)? a. Older adult with COPD b. Middle-aged client receiving a blood transfusion c. Older adult who has aspirated his tube feeding d. Young adult with a broken leg from a motorcycle accident

c. Older adult who has aspirated his tube feeding

The registered nurse is assigning a practical nurse to care for a client who has leukemia. Which instruction does the registered nurse provide to the practical nurse when delegating this client's care? a. Evaluate the amount of protein the client eats. b. Assess the client's roommate for symptoms of infection. c. Perform effective hand hygiene frequently. d. Wear a mask when entering the room.

c. Perform effective hand hygiene frequently.

What fluid is the best choice when a child with mucositis asks for something to drink? a. Hot chocolate b. Lemonade c. Popsicle d. Orange juice

c. Popsicle

In clients with any type of acid-base imbalance, the nurse places the priority on monitoring which electrolyte? a. Sodium b. Calcium c. Potassium d. Magnesium

c. Potassium

The nurse prioritizes which intervention in a client with xerostomia secondary to radiation therapy to the neck area? a. Applying lotions and oils to affected areas b. Wearing a hat to decrease heat loss c. Providing oral care after meals and at bedtime d. Monitoring vital signs every 4 hours

c. Providing oral care after meals and at bedtime

The nurse assesses a client who has a hemothorax and a chest tube inserted on the right side. What finding requires immediate attention? a. Pain at the chest tube insertion site b. Fluctuation in the water seal chamber with breathing c. Puffiness of the skin around the chest tube insertion site and a crackling feeling d. Dullness to percussion on the affected side

c. Puffiness of the skin around the chest tube insertion site and a crackling feeling

The nurse is assessing a patient and finds crackles in the lung bases and neck vein distention. Which action will the nurse take first? a. Offer calcium-rich foods. b. Administer diuretic. c. Raise head of bed. d. Increase fluids.

c. Raise head of bed.

A nurse is caring for a cancer patient who presents with anorexia, blood pressure 100/60, and elevated white blood cell count. Which primary purpose for starting total parenteral nutrition (TPN) will the nurse add to the care plan? a. Stimulate the patient's appetite to eat. b. Deliver antibiotics to fight off infection. c. Replace fluid, electrolytes, and nutrients. d. Provide medication to raise blood pressure.

c. Replace fluid, electrolytes, and nutrients.

The nurse assesses for acidosis in the client with which assessment data? a. Serum sodium level of 130 mEq per liter and peripheral edema b. Serum sodium level of 144 mEq per liter and tachycardia c. Serum potassium level of 6.5 mEq per liter and flaccid paralysis d. Serum potassium level of 4.5 mEq per liter and hyperactive deep tendon reflexes

c. Serum potassium level of 6.5 mEq per liter and flaccid paralysis

The nurse is evaluating the effectiveness of the intravenous fluid therapy in a patient with hypernatremia. Which finding indicates goal achievement? a. Urine output increases to 150 mL/hr. b. Systolic and diastolic blood pressure decreases. c. Serum sodium concentration returns to normal. d. Large amounts of emesis and diarrhea decrease.

c. Serum sodium concentration returns to normal.

When planning for cancer survivor care needs, which information should the nurse consider? a. Survivorship care plans are reviewed with the patient at home. b. All health care agencies provide survivorship care plans. c. Some survivors are discharged with no survivor plan. d. The plan does not deal with future cancer screenings.

c. Some survivors are discharged with no survivor plan.

The nurse is caring for an adult patient in the clinic who has been evacuated and is a victim of flooding. The nurse teaches the patient about rest, exercise, and eating properly and how to utilize deep breathing and visualization. What is the primary rationale for the nurse's actions related to the teaching? a. Topics taught are standard information taught during health care visits. b. The patient requested this information to teach the extended family members. c. Stress for long periods of time can lead to exhaustion and decreased resistance to infection. d. These techniques will help the patient manage the pain and loss of personal belongings.

c. Stress for long periods of time can lead to exhaustion and decreased resistance to infection.

Which intervention is appropriate for a child receiving high doses of steroids? a. Limit activity and receive home schooling. b. Decrease the amount of potassium in the diet. c. Substitute a killed virus vaccine for live virus vaccines. d. Monitor for seizure activity.

c. Substitute a killed virus vaccine for live virus vaccines.

In evaluating the electrocardiogram (ECG) in a client with acidosis, the nurse correlates which ECG change with effectiveness of therapy? a. Small U-waves present after each complex b. Heart rate decreased to 62 beats/min c. T-waves present, normal height d. P-wave preceding the QRS complex

c. T-waves present, normal height

The nurse is interested in primary prevention for cancer. Which activity does the nurse most likely participate in? a. Distributing occult fecal blood test kits to people at the shopping mall b. Arranging transportation volunteers for clients undergoing radiation therapy c. Teaching high school students the dangers of using tobacco d. Educating adolescent girls about getting an annual Papanicolaou (PAP) smear

c. Teaching high school students the dangers of using tobacco

The nurse is caring for a patient who is susceptible to infection. Which instruction will the nurse include in an educational session to decrease the risk of infection? a. Teaching the patient about fall prevention b. Teaching the patient to take a temperature c. Teaching the patient to select nutritious foods d. Teaching the patient about the effects of alcohol

c. Teaching the patient to select nutritious foods

The nurse should base a response to a parent's question about the prognosis of acute lymphoblastic leukemia (ALL) on the knowledge that a. Leukemia is a fatal disease, although chemotherapy provides increasingly longer periods of remission. b. Research to find a cure for childhood cancers is very active. c. The majority of children go into remission and remain symptom free when treatment is completed. d. It usually takes several months of chemotherapy to achieve a remission.

c. The majority of children go into remission and remain symptom free when treatment is completed.

What should the nurse include in a teaching plan for the mother of a toddler who will be taking prednisone for several months? a. The medication should be taken between meals. b. The medication needs to be discontinued because of the risks associated with long-term usage. c. The medication should not be stopped abruptly. d. The medication may lower blood glucose, so the mother needs to observe for signs of hypoglycemia.

c. The medication should not be stopped abruptly.

In reviewing the pathophysiology of a particular type of cancer, the nurse correlates the generation time for cancer development with which description? a. The rate at which cancer cells are able to migrate and metastasize to different sites b. How long it takes for a malignant tumor to double in size by mitotic cell divisions c. The period of time needed for one cell to divide into two cells by mitosis d. The period of time between cell damage and expression of a malignancy

c. The period of time needed for one cell to divide into two cells by mitosis

The nurse includes which information about benign tumors when presenting an in-service on cancer? a. They can wander far throughout the body. b. They are smaller than 2 cm. c. They retain a small nuclear-to-cytoplasmic ratio. d. They look different from the tissue they arose from.

c. They retain a small nuclear-to-cytoplasmic ratio.

What assessment should the nurse make before initiating an intravenous (IV) infusion of dextrose 5% in 0.9% normal saline solution with 10 mEq of potassium chloride for a child hospitalized with dehydration? a. Fluid intake b. Number of stools c. Urine output d. Capillary refill

c. Urine output

The nurse is caring for a patient in an intensive care unit who needs a bath. Which priority action will the nurse take to decrease the potential for a health care-associated infection? a. Use local anesthetic on reddened areas. b. Use nonallergenic tape on dressings. c. Use a chlorhexidine wash. d. Use filtered water.

c. Use a chlorhexidine wash.

The nurse is caring for a patient with an incision. Which actions will best indicate an understanding of medical and surgical asepsis for a sterile dressing change? a. Donning clean goggles, gown, and gloves to dress the wound b. Donning sterile gown and gloves to remove the wound dressing c. Utilizing clean gloves to remove the dressing and sterile supplies for the new dressing d. Utilizing clean gloves to remove the dressing and clean supplies for the new dressing

c. Utilizing clean gloves to remove the dressing and sterile supplies for the new dressing

The health care provider asks the nurse to monitor the fluid volume status of a heart failure patient and a patient at risk for clinical dehydration. Which is the most effective nursing intervention for monitoring both of these patients? a. Assess the patients for edema in extremities. b. Ask the patients to record their intake and output. c. Weigh the patients every morning before breakfast. d. Measure the patients' blood pressures every 4 hours.

c. Weigh the patients every morning before breakfast.

The nurse interprets which arterial blood gas values as partially compensated metabolic acidosis? a. pH 7.28, HCO3- 19 mEq/L, PCO2 45 mm Hg, PO2 96 mm Hg b. pH 7.45, HCO3- 22 mEq/L, PCO2 40 mm Hg, PO2 98 mm Hg c. pH 7.32, HCO3- 17 mEq/L, PCO2 25 mm Hg, PO2 98 mm Hg d. pH 7.48, HCO3- 28 mEq/L, PCO2 45 mm Hg, PO2 92 mm Hg

c. pH 7.32, HCO3- 17 mEq/L, PCO2 25 mm Hg, PO2 98 mm Hg

A client has respiratory acidosis. The nurse evaluates that treatment is being effective with which arterial blood gas values? a. pH 7.28, HCO3- 12 mEq/L, PCO2 45 mm Hg, PO2 96 mm Hg b. pH 7.32, HCO3- 17 mEq/L, PCO2 25 mm Hg, PO2 98 mm Hg c. pH 7.35, HCO3- 36 mEq/L, PCO2 65 mm Hg, PO2 78 mm Hg d. pH 7.48, HCO3- 12 mEq/L, PCO2 35 mm Hg, PO2 85 mm Hg

c. pH 7.35, HCO3- 36 mEq/L, PCO2 65 mm Hg, PO2 78 mm Hg

The patient has type B chronic obstructive pulmonary disease (COPD) exacerbated by an acute upper respiratory infection. Which blood gas values should the nurse expect to see? a. pH high, PaCO2 high, HCO3- high b. pH low, PaCO2 low, HCO3- low c. pH low, PaCO2 high, HCO3- high d. pH low, PaCO2 high, HCO3- normal

c. pH low, PaCO2 high, HCO3- high

While reviewing the complete blood count (CBC) of a patient on her unit, the nurse notes elevated basophil and eosinophil readings. The nurse realizes that this is most indicative of a _____ infection. a. bacterial b. fungal c. parasitic d. viral

c. parasitic

The nurse is working on a plan of care with her patient which includes turning and positioning and adequate nutrition to help the patient maintain intact skin integrity. The nurse helps the patient to realize that this breaks the chain of infection by eliminating a a. host. b. mode of transmission. c. portal of entry. d. reservoir.

c. portal of entry.

The caretaker of a confused client with functional incontinence asks about having an in-dwelling catheter placed. Which is the nurse's best response? a. "You must be very aggravated about this situation. I will call the provider with this request." b. "I will teach you how to insert the catheter, which should be used just at night." c. "We can teach you how to perform intermittent catheterization to drain the bladder." d. "Because the client is confused, we need to place priority on keeping the skin clean and dry."

d. "Because the client is confused, we need to place priority on keeping the skin clean and dry."

A client has functional urinary incontinence. Which instruction by the nurse to the client and family helps meet an expected outcome for this condition? a. "You must clean around your catheter daily with soap and water." b. "Wash the vaginal weights with a 10% bleach solution after each use." c. "Operations to repair your bladder are available, and you can consider these." d. "Buy slacks with elastic waistbands that are easy to pull down."

d. "Buy slacks with elastic waistbands that are easy to pull down."

A middle-aged client is having a physical examination and is worried about cancer risk. Which question is most important for the nurse to ask? a. "How much time do you spend in the sun?" b. "How many servings of fruits and vegetables do you eat every day?" c. "How often do you eat processed meats like bologna?" d. "Do you smoke cigarettes or have you ever used tobacco products?"

d. "Do you smoke cigarettes or have you ever used tobacco products?"

A chemotherapy patient has gained 5 pounds in 2 days. Which assessment question by the nurse is most appropriate? a. "Are you following any weight loss program?" b. "How many calories a day do you consume?" c. "Do you have dry mouth or feel thirsty?" d. "How many times a day do you urinate?"

d. "How many times a day do you urinate?"

The nurse has telephone messages from four patients who requested information and assistance. Which one should the nurse refer to a social worker or community agency first? a. "Is there a place that I can dispose of my unused morphine pills?" b. "I want to lose at least 20 pounds without getting sick this time." c. "I think I have asthma because I cough when dogs are near." d. "I ran out of money and am cutting my insulin dose in half."

d. "I ran out of money and am cutting my insulin dose in half."

What statement indicates that the client understands teaching about neutropenia? a. "I need to use a soft toothbrush." b. "I have to wear a mask at all times." c. "My grandchildren may get an infection from me." d. "I will call my doctor if I have an increase in temperature."

d. "I will call my doctor if I have an increase in temperature."

Which comment made by a client with breast cancer indicates a need for clarification regarding cancer causes and prevention? a. "I will eat a low-fat, high-fiber diet from now on." b. "Probably nothing I did or didn't do caused this cancer." c. "I hope my daughter doesn't develop breast cancer." d. "Regular mammograms on my other breast will prevent cancer."

d. "Regular mammograms on my other breast will prevent cancer."

The patient has lung cancer and voices concerns about cancer treatments affecting sexuality. What is the nurse's best reply? a. "That is something to ask the health care provider." b. "Chemotherapy will work in the lungs and should have no effect on sexuality." c. "How cancer treatment affects sexuality depends on how active you are and your age." d. "Sexual changes are common with cancer therapy. Let me get someone who can answer your questions."

d. "Sexual changes are common with cancer therapy. Let me get someone who can answer your questions."

A client receiving intravenous chemotherapy asks the nurse the reason for wearing a mask, gloves, and gown while administering drugs to the client. What is the nurse's best response? a. "These coverings protect you from getting an infection from me." b. "I am preventing the spread of infection from you to me or any other client here." c. "The policy is for any nurse giving these drugs to wear a gown, gloves, and mask." d. "The clothing protects me from accidentally absorbing these drugs."

d. "The clothing protects me from accidentally absorbing these drugs."

How should the nurse explain positioning for a lumbar puncture to a 5-year-old child? a. "You will be on your knees with your head down on the table." b. "You will be able to sit up with your chin against your chest." c. "You will be on your side with the head of your bed slightly raised." d. "You will lie on your side and bend your knees so that they touch your chin."

d. "You will lie on your side and bend your knees so that they touch your chin."

Which statement, if made by a nurse to the parents of a child with leukemia, indicates an understanding of teaching related to home care associated with the disease? a. "Your son's blood pressure must be taken daily while he is on chemotherapy." b. "Limit your son's fluid intake just in case he has central nervous system involvement." c. "Your son must receive all of his immunizations in a timely manner." d. "Your son's temperature should be taken frequently."

d. "Your son's temperature should be taken frequently."

The nurse is preparing a client with leukemia for a peripheral stem cell transfusion. Which information does the nurse provide the client? a. "Nausea and vomiting are common after the transfusion." b. "The transfusion will take about 6 hours." c. "You may have numbness in your fingers and toes." d. "Your urine may be red for a short time."

d. "Your urine may be red for a short time."

The nurse monitors for which acid-base problem in a client who is taking furosemide (Lasix) for hypertension? a. Acid excess secondary to respiratory acidosis b. Acid deficit secondary to respiratory alkalosis c. Acid excess secondary to metabolic acidosis d. Acid deficit secondary to metabolic alkalosis

d. Acid deficit secondary to metabolic alkalosis

Hematopoietic stem cell transplantation (HSCT) is the standard treatment for a child in his or her first remission with what cancer? a. ALL b. Non-Hodgkin lymphoma c. Wilms' tumor d. Acute myeloblastic leukemia (AML)

d. Acute myeloblastic leukemia (AML)

The intensive care nurse is caring for an intubated client who has severe sepsis that led to acute respiratory distress. Which nursing intervention is most appropriate during this stage of sepsis? a. Check blood glucose levels every 4 hours. b. Monitor intake and urinary output twice each shift. c. Decrease ventilator rate and tidal volume. d. Administer prescribed low-dose corticosteroids.

d. Administer prescribed low-dose corticosteroids.

While the nurse is obtaining the health history of a 75-year-old female patient, which of the following has the greatest implication for the development of cancer? a. Being a 75-year-old woman b. Family history of hypertension c. Cigarette smoking as a teenager d. Advancing age

d. Advancing age

A client has kidney stones from secondary hyperoxaluria. Which medication does the nurse anticipate administering? a. Phenazopyridine (Pyridium) b. Propantheline (Pro-Banthine) c. Tolterodine (Detrol-LA) d. Allopurinol (Zyloprim)

d. Allopurinol (Zyloprim)

A client is admitted owing to difficulty breathing. The nurse assesses the client's color, lung sounds, and pulse oximetry reading. The pulse oximetry is 90%. What is the nurse's next action? a. Give an intermittent positive-pressure breathing treatment. b. Administer a rescue inhaler. c. Call for a chest x-ray. d. Assess an arterial blood gas.

d. Assess an arterial blood gas.

The nurse notes that each time the mechanical ventilator delivers a breath to a client with acute respiratory distress syndrome (ARDS), the peak inspiratory pressure alarm sounds. What is the nurse's best intervention? a. Suction the client. b. Perform chest physiotherapy. c. Administer an inhaler. d. Assess the airway.

d. Assess the airway.

The client receiving mechanical ventilation has become more restless over the course of the shift. Which is the nurse's first action? a. Sedate the client. b. Call the health care provider. c. Assess the client for pain. d. Assess the client's oxygenation.

d. Assess the client's oxygenation.

A client with dyspnea is becoming very anxious. An arterial blood gas (ABG) shows a PaO2 of 93 mm Hg. How does the nurse best intervene? a. Increase the oxygen. b. Administer an antianxiety medication. c. Administer a bronchodilator. d. Assist with relaxation techniques.

d. Assist with relaxation techniques.

The nurse is caring for a patient with hyperkalemia. Which body system assessment is the priority? a. Gastrointestinal b. Neurological c. Respiratory d. Cardiac

d. Cardiac

After receiving change-of-shift report, which client does the nurse assess first? a. Client with leukemia who needs an antiemetic before chemotherapy b. Client with breast cancer scheduled for external beam radiation c. Client with xerostomia associated with laryngeal cancer d. Client with neutropenia who has just been admitted with a possible infection

d. Client with neutropenia who has just been admitted with a possible infection

The nurse is caring for a client who has been intubated and placed on a ventilator for treatment of acute respiratory distress syndrome (ARDS). Aside from assessing oxygenation, what is the nurse's priority action? a. Assess hemoglobin. b. Administer ferrous sulfate. c. Assess muscle strength. d. Consult with the registered dietitian.

d. Consult with the registered dietitian.

A nurse is caring for a patient who is receiving peripheral intravenous (IV) therapy. When the nurse is flushing the patient's peripheral IV, the patient reports pain. Upon assessment, the nurse notices a red streak that is warm to the touch. What is the nurse's initial action? a. Record a phlebitis grade of 4. b. Assign an infiltration grade. c. Apply moist compress. d. Discontinue the IV.

d. Discontinue the IV.

The nurse is caring for a client receiving heparin and warfarin therapy for a pulmonary embolus. The client's international normalized ratio (INR) is 2.0. What is the nurse's best action? a. Increase the heparin dose. b. Increase the warfarin dose. c. Continue the current therapy. d. Discontinue the heparin.

d. Discontinue the heparin.

A diabetic patient presents to the clinic for a dressing change. The wound is located on the right foot and has purulent yellow drainage. Which action will the nurse take to prevent the spread of infection? a. Position the patient comfortably on the stretcher. b. Explain the procedure for dressing change to the patient. c. Review the medication list that the patient brought from home. d. Don gloves and other appropriate personal protective equipment.

d. Don gloves and other appropriate personal protective equipment.

Adjuvant treatment with tamoxifen may be recommended for patients with breast cancer if the tumor is a. Smaller than 5 cm b. Located in the upper outer quadrant only c. Contained only in the breast d. Estrogen receptive

d. Estrogen receptive

What information should the nurse teach workers at a daycare center about RSV? a. RSV is transmitted through particles in the air. b. RSV can live on skin or paper for up to a few seconds after contact. c. RSV can survive on nonporous surfaces for about 60 minutes. d. Frequent handwashing can decrease the spread of the virus.

d. Frequent handwashing can decrease the spread of the virus.

A client has late-stage colon cancer with metastasis to the spine and bones. Which nursing intervention does the nurse add to the care plan to address a priority problem? a. Provide six small meals and snacks daily. b. Offer the client prune juice twice a day. c. Ensure that the client gets adequate rest. d. Give the client pain medications around the clock.

d. Give the client pain medications around the clock.

A nurse is working on a cancer unit. The unit uses the National Coalition for Cancer Survivorship definition for a cancer survivor. Which definition will the nurse use? a. Been cancer free for 5 years after diagnosis b. Been cancer free for 3 years after diagnosis c. Had cancer and is declared cancer free d. Had cancer and extends until death

d. Had cancer and extends until death

Which assessment finding of a client requires the nurse's immediate action? a. Being intubated for 4 days b. Uneven breath sounds c. Wheezing on auscultation d. Having the endotracheal (ET) tube taped to the lower jaw

d. Having the endotracheal (ET) tube taped to the lower jaw

The nurse counsels a woman who has a BRCA1 gene that she has what chance for developing breast cancer during her lifetime? a. None; this gene has a protective effect b. Same as the general population c. Lower than the general population d. Higher than the general population

d. Higher than the general population

A child has a 2-day history of vomiting and diarrhea. He has hypoactive bowel sounds and an irregular pulse. Electrolyte values are sodium, 139 mEq/L; potassium, 3.3 mEq/L; and calcium, 9.5 mg/dL. This child is likely to have which of the following electrolyte imbalances? a. Hyponatremia b. Hypocalcemia c. Hyperkalemia d. Hypokalemia

d. Hypokalemia

The nurse is caring for a client who is taken off a ventilator and placed on continuous positive airway pressure (CPAP). What intervention is most appropriate for this client? a. Administering antianxiety medications PRN b. Administering a medication to help the client sleep c. Telling the client to relax and let the ventilator do the work d. Making sure the client is breathing spontaneously

d. Making sure the client is breathing spontaneously

Malignant cell growth is uncontrolled because of which action? a. Cancer cells always divide more rapidly than normal cells. b. Mitosis of malignant cells usually produces more than two daughter cells. c. Malignant cells bypass one or more phases of the cell cycle during cell division. d. Malignant cells enter the cell cycle frequently, making cell division continuous.

d. Malignant cells enter the cell cycle frequently, making cell division continuous.

What is the priority nursing intervention for a 6-month-old infant hospitalized with diarrhea and dehydration? a. Estimating insensible fluid loss b. Collecting urine for culture and sensitivity c. Palpating the posterior fontanel d. Measuring the infant's weight

d. Measuring the infant's weight

The nurse is caring for a patient who received a bone marrow transplant 10 days ago. The nurse would monitor for which of the following clinical manifestations that could indicate a potentially life-threatening situation? a. Mucositis b. Confusion c. Depression d. Mild temperature elevation

d. Mild temperature elevation

Why do infants and young children quickly have respiratory distress in acute and chronic alterations of the respiratory system? a. They have a widened, shorter airway. b. There is a defect in their sucking ability. c. The gag reflex increases mucus production. d. Mucus and edema obstruct small airways.

d. Mucus and edema obstruct small airways.

A client with a large pulmonary embolism is receiving alteplase (Activase). The nurse notes frank red blood in the Foley catheter drainage bag. What is the nurse's first action? a. Irrigate the Foley. b. Administer an antibiotic. c. Clamp the Foley. d. Notify the health care provider.

d. Notify the health care provider.

What is the most appropriate nursing action when the nurse notes a reddened area on the forearm of a neutropenic child with leukemia? a. Massage the area. b. Turn the child more frequently. c. Document the finding and continue to observe the area. d. Notify the physician.

d. Notify the physician.

A nurse is assessing a cancer survivor for chemotherapy-induced peripheral neurotoxicity (CPIN). Which assessment finding is consistent with CPIN? a. Hearing loss b. Devastating depression c. Extreme loss of motor functioning d. Numbness and tingling in hands and feet

d. Numbness and tingling in hands and feet

The nurse assesses which client most carefully for cancer development? a. Young man receiving radiation therapy for a brain tumor b. Young adult woman who recently had postpartum hemorrhage c. Adolescent male recently diagnosed with acquired immune deficiency syndrome (AIDS) d. Older woman undergoing chemotherapy for bowel cancer

d. Older woman undergoing chemotherapy for bowel cancer

The nurse assesses the client with which condition most carefully for the risk of developing acute respiratory acidosis? a. Allergic rhinitis and sinusitis on sulfa antibiotics b. Type 1 diabetes and urinary tract infection c. Emphysema and undergoing nasogastric (NG) tube suctioning d. On patient-controlled analgesia after abdominal surgery

d. On patient-controlled analgesia after abdominal surgery

The nurse is assessing a client who was admitted for treatment of shock. Which manifestation indicates that the client's shock is caused by sepsis? a. Hypotension b. Pale clammy skin c. Anxiety and confusion d. Oozing of blood at the IV site

d. Oozing of blood at the IV site

The nurse is caring for a patient diagnosed with cancer. The family of the patient asks the nurse for resources about the cancer. What should the nurse do? a. Refer family members to the health care provider. b. Inform them that few options are available. c. Maintain confidentiality by keeping silent. d. Provide the family with the information.

d. Provide the family with the information.

The nurse who is teaching a group of women about breast cancer should tell the women that a. Risk factors identify almost all women who will develop breast cancer. b. African-American women have a higher rate of breast cancer. c. One in 10 women in the United States will develop breast cancer in her lifetime. d. The exact cause of breast cancer is unknown.

d. The exact cause of breast cancer is unknown.

A cancer survivor is in the intensive care unit (ICU). Some of the patient's family is from out of town and would like to see the patient even though it is not "official" visiting hours. The patient is anxious to see family members. The nurse allows the family to visit. What is the rationale for the nurse's actions? a. The nurse disagrees with the established time for visiting. b. The nurse realizes that the patient is dying. c. The nurse feels there is no real reason to have limited visiting hours. d. The nurse believes that the visit will help relieve psychological stress.

d. The nurse believes that the visit will help relieve psychological stress.

Which organs and tissues control the two types of specific immune functions? a. The spleen and mucous membranes b. Upper and lower intestinal lymphoid tissue c. The skin and lymph nodes d. The thymus and bone marrow

d. The thymus and bone marrow

The nurse is caring for a patient who has successfully undergone cancer therapy and will be discharged home soon. The patient is concerned about going home and not knowing what to do. Which information is the most valuable for the nurse to share with the patient? a. The nurse will develop a plan of care that will tell exactly what needs to be done. b. If any issues arise, call the health care provider and follow the instructions. c. Proper cancer treatment has been provided, and nothing else is required. d. There is a team that will provide support and care that may be needed.

d. There is a team that will provide support and care that may be needed.

Which assessment finding will the nurse expect for a patient with the following laboratory values: sodium 145 mEq/L, potassium 4.5 mEq/L, calcium 4.5 mg/dL? a. Weak quadriceps muscles b. Decreased deep tendon reflexes c. Light-headedness when standing up d. Tingling of extremities with possible tetany

d. Tingling of extremities with possible tetany

A nurse is assisting the health care provider in inserting a central line. Which action indicates the nurse is following the recommended bundle protocol to reduce central line-associated bloodstream infections (CLABSI)? a. Preps skin with povidone-iodine solution. b. Suggests the femoral vein for insertion site. c. Applies double gloving without hand hygiene. d. Uses chlorhexidine skin antisepsis prior to insertion.

d. Uses chlorhexidine skin antisepsis prior to insertion.

A client has overflow incontinence. Which intervention does the nurse add to this client's care plan to assist with elimination? a. Stroking the medial aspect of the thigh b. Using intermittent catheterization c. Providing digital anal stimulation d. Using the Valsalva maneuver

d. Using the Valsalva maneuver

The nurse is providing an educational session for a group of preschool workers. The nurse reminds the group about the most important thing to do to prevent the spread of infection. Which information did the nurse share with the preschool workers? a. Encourage preschool children to eat a nutritious diet. b. Suggest that parents provide a multivitamin to the children. c. Clean the toys every afternoon before putting them away. d. Wash their hands between each interaction with children.

d. Wash their hands between each interaction with children.

The nurse is caring for a patient who is at risk for infection. Which action by the nurse indicates correct understanding about standard precautions? a. Teaches the patient about good nutrition b. Dons gloves when wearing artificial nails c. Disposes an uncapped needle in the designated container d. Wears eyewear when emptying the urinary drainage bag

d. Wears eyewear when emptying the urinary drainage bag

A true statement describing the differences in the pediatric genitourinary system compared with the adult genitourinary system is that a. The young infant's kidneys can more effectively concentrate urine than an adult's kidneys. b. After 6 years of age, kidney function is nearly like that of an adult. c. Unlike adults, most children do not regain normal kidney function after acute renal failure. d. Young children have shorter urethras, which can predispose them to UTIs.

d. Young children have shorter urethras, which can predispose them to UTIs.

The nurse is assessing arterial blood gases (ABGs). The client with which ABG reading requires the nurse's immediate attention? a. pH, 7.32; PaCO2, 55 mm Hg; PaO2, 70 mm Hg b. pH, 7.45; PaCO2, 42 mm Hg; PaO2, 70 mm Hg c. pH, 7.48; PaCO2, 38 mm Hg; PaO2, 60 mm Hg d. pH, 7.55; PaCO2, 32 mm Hg; PaO2, 50 mm Hg

d. pH, 7.55; PaCO2, 32 mm Hg; PaO2, 50 mm Hg

In order to provide the best intervention for a patient, the nurse is often responsible for obtaining a sample of exudate for culture. This test will identify a. whether a patient has an infection. b. where an infection is located. c. what cells are being utilized by the body to attack an infection. d. what specific type of pathogen is causing an infection.

d. what specific type of pathogen is causing an infection.

A child with secondary enuresis who complains of dysuria or urgency should be evaluated for what condition? Select all that apply. a. Hypocalciuria b. Nephrotic syndrome c. Glomerulonephritis d.UTI e. Diabetes mellitus

d.UTI e. Diabetes mellitus


Kaugnay na mga set ng pag-aaral

Metro Manila and its Literature

View Set

Chapter 5: Gross Income and Exclusions

View Set

Residential Construction and the Appraiser

View Set

Chapter 15: Influence, Empowerment, and Politics

View Set

Routes, Emergency Action Plans, Fire Protection Plans, and Fire Protection OSHA 10 hour General Industry Exits

View Set

Antepartum, Reproduction and Sexuality Unit #2

View Set

BIOL1101 - evolution and biological diversity

View Set

Skeletal System - Arms and Shoulder and Hand Bones

View Set